Using only 1s, make 29 with the minimum number of digits$tau$ is greatest! $tau$ is all (from 1 to 20)Create...

PGF Plot settings

Overfitting and Underfitting

ArcとVecのmutableエラー'cannot borrow as mutable'について

Superposition of light waves of different colors

How long would gestation period be for dwarfs?

Is it a fallacy if someone claims they need an explanation for every word of your argument to the point where they don't understand common terms?

It took me a lot of time to make this, pls like. (YouTube Comments #1)

What is the purpose of easy combat scenarios that don't need resource expenditure?

Why did Bush enact a completely different foreign policy to that which he espoused during the 2000 Presidential election campaign?

High pressure canisters of air as gun-less projectiles

What kind of hardware implements Fourier transform?

SQL Create Script DDL Table from View

Copy large no of files of specific date to another directory?

Can you enable Unicorn NewItemsOnly for specific folders?

What do you call a fact that doesn't match the settings?

Using only 1s, make 29 with the minimum number of digits

Two functions in the same line

A starship is travelling at 0.9c and collides with a small rock. Will it leave a clean hole through, or will more happen?

Numbers of Exaggeration in Talmud

Can a hotel cancel a confirmed reservation?

The terminology for an excluded solution

Called into a meeting and told we are being made redundant (laid off) and "not to share outside". Can I tell my partner?

Slicing User Stories

If I delete my router's history can my ISP still provide it to my parents?



Using only 1s, make 29 with the minimum number of digits


$tau$ is greatest! $tau$ is all (from 1 to 20)Create integers from 1 to 50 using only one integer and other functionsMaking π from 1 2 3 4 5 6 7 8 9Using the digits 2, 3, 4 make an expression for 30Express the number $2015$ using only the digit $2$ twiceIterative Floors and CeilingsMaking 4 with 4 ones (with a twist!)Make 2008 from Φ (Golden Ratio)Use 2, 0, 1 and 8 to make 71Create all numbers from 1-100 by using 1,3,3,7













26












$begingroup$



Use the minimum number of ones to make 29.




Here is the list of operations permitted:




  • "Standard" operations, such as: $x+y$, $x-y$, $xtimes y$, $xdiv y$

  • Negation: $-x$

  • Exponentiation of two numbers: $x^y$

  • The square root of a number: $sqrt{x}$

  • The factorial: $x!$

  • Concatenation of the original digits: $x|x$, $x|x|x$, $x|x|x$, etc. This means that you cannot concatenate like this: $(1+1)|1 = 2|1 = 21$


You can only use ones, and the result must be exactly $29$, not "about" 29.



You may not use any other operations. That includes $log{x}$, $left lfloor{x}right rfloor$, $left lceil{x}right rceil$ and any others. You must only use base 10, and you not use any decimal points (i.e. no .1, .11, etc.).



The record to beat is 7.










share|improve this question









New contributor




Allan Cao is a new contributor to this site. Take care in asking for clarification, commenting, and answering.
Check out our Code of Conduct.







$endgroup$








  • 10




    $begingroup$
    (1+1+1) / .1 - 1
    $endgroup$
    – shoover
    Feb 28 at 6:28






  • 2




    $begingroup$
    Do you know if there is a solution with less than 7 digits, or are we just hoping there is one?
    $endgroup$
    – luchonacho
    Feb 28 at 11:01






  • 2




    $begingroup$
    @AllanCao Are you, by any chance, a smurf account of one of the regulars? Never have I ever seen such a well-defined formation-of-numbers puzzle with the "new contributor" flag on it. Great job, and welcome to PSE!
    $endgroup$
    – Bass
    Feb 28 at 15:31






  • 1




    $begingroup$
    Thank you! I'm not an alt account or anything. As well, I think 6 might be possible but I haven't found a solution. I've only gone down to 7 which I found to be a fun puzzle so I decided to share it.
    $endgroup$
    – Allan Cao
    2 days ago






  • 2




    $begingroup$
    I've got a 5-er too... By thinking out of the box quite a bit... Would be fun to post - but not enough rep...
    $endgroup$
    – ppeterka
    yesterday
















26












$begingroup$



Use the minimum number of ones to make 29.




Here is the list of operations permitted:




  • "Standard" operations, such as: $x+y$, $x-y$, $xtimes y$, $xdiv y$

  • Negation: $-x$

  • Exponentiation of two numbers: $x^y$

  • The square root of a number: $sqrt{x}$

  • The factorial: $x!$

  • Concatenation of the original digits: $x|x$, $x|x|x$, $x|x|x$, etc. This means that you cannot concatenate like this: $(1+1)|1 = 2|1 = 21$


You can only use ones, and the result must be exactly $29$, not "about" 29.



You may not use any other operations. That includes $log{x}$, $left lfloor{x}right rfloor$, $left lceil{x}right rceil$ and any others. You must only use base 10, and you not use any decimal points (i.e. no .1, .11, etc.).



The record to beat is 7.










share|improve this question









New contributor




Allan Cao is a new contributor to this site. Take care in asking for clarification, commenting, and answering.
Check out our Code of Conduct.







$endgroup$








  • 10




    $begingroup$
    (1+1+1) / .1 - 1
    $endgroup$
    – shoover
    Feb 28 at 6:28






  • 2




    $begingroup$
    Do you know if there is a solution with less than 7 digits, or are we just hoping there is one?
    $endgroup$
    – luchonacho
    Feb 28 at 11:01






  • 2




    $begingroup$
    @AllanCao Are you, by any chance, a smurf account of one of the regulars? Never have I ever seen such a well-defined formation-of-numbers puzzle with the "new contributor" flag on it. Great job, and welcome to PSE!
    $endgroup$
    – Bass
    Feb 28 at 15:31






  • 1




    $begingroup$
    Thank you! I'm not an alt account or anything. As well, I think 6 might be possible but I haven't found a solution. I've only gone down to 7 which I found to be a fun puzzle so I decided to share it.
    $endgroup$
    – Allan Cao
    2 days ago






  • 2




    $begingroup$
    I've got a 5-er too... By thinking out of the box quite a bit... Would be fun to post - but not enough rep...
    $endgroup$
    – ppeterka
    yesterday














26












26








26


4



$begingroup$



Use the minimum number of ones to make 29.




Here is the list of operations permitted:




  • "Standard" operations, such as: $x+y$, $x-y$, $xtimes y$, $xdiv y$

  • Negation: $-x$

  • Exponentiation of two numbers: $x^y$

  • The square root of a number: $sqrt{x}$

  • The factorial: $x!$

  • Concatenation of the original digits: $x|x$, $x|x|x$, $x|x|x$, etc. This means that you cannot concatenate like this: $(1+1)|1 = 2|1 = 21$


You can only use ones, and the result must be exactly $29$, not "about" 29.



You may not use any other operations. That includes $log{x}$, $left lfloor{x}right rfloor$, $left lceil{x}right rceil$ and any others. You must only use base 10, and you not use any decimal points (i.e. no .1, .11, etc.).



The record to beat is 7.










share|improve this question









New contributor




Allan Cao is a new contributor to this site. Take care in asking for clarification, commenting, and answering.
Check out our Code of Conduct.







$endgroup$





Use the minimum number of ones to make 29.




Here is the list of operations permitted:




  • "Standard" operations, such as: $x+y$, $x-y$, $xtimes y$, $xdiv y$

  • Negation: $-x$

  • Exponentiation of two numbers: $x^y$

  • The square root of a number: $sqrt{x}$

  • The factorial: $x!$

  • Concatenation of the original digits: $x|x$, $x|x|x$, $x|x|x$, etc. This means that you cannot concatenate like this: $(1+1)|1 = 2|1 = 21$


You can only use ones, and the result must be exactly $29$, not "about" 29.



You may not use any other operations. That includes $log{x}$, $left lfloor{x}right rfloor$, $left lceil{x}right rceil$ and any others. You must only use base 10, and you not use any decimal points (i.e. no .1, .11, etc.).



The record to beat is 7.







mathematics calculation-puzzle formation-of-numbers






share|improve this question









New contributor




Allan Cao is a new contributor to this site. Take care in asking for clarification, commenting, and answering.
Check out our Code of Conduct.











share|improve this question









New contributor




Allan Cao is a new contributor to this site. Take care in asking for clarification, commenting, and answering.
Check out our Code of Conduct.









share|improve this question




share|improve this question








edited yesterday









Hugh

2,27511126




2,27511126






New contributor




Allan Cao is a new contributor to this site. Take care in asking for clarification, commenting, and answering.
Check out our Code of Conduct.









asked Feb 28 at 2:18









Allan CaoAllan Cao

235311




235311




New contributor




Allan Cao is a new contributor to this site. Take care in asking for clarification, commenting, and answering.
Check out our Code of Conduct.





New contributor





Allan Cao is a new contributor to this site. Take care in asking for clarification, commenting, and answering.
Check out our Code of Conduct.






Allan Cao is a new contributor to this site. Take care in asking for clarification, commenting, and answering.
Check out our Code of Conduct.








  • 10




    $begingroup$
    (1+1+1) / .1 - 1
    $endgroup$
    – shoover
    Feb 28 at 6:28






  • 2




    $begingroup$
    Do you know if there is a solution with less than 7 digits, or are we just hoping there is one?
    $endgroup$
    – luchonacho
    Feb 28 at 11:01






  • 2




    $begingroup$
    @AllanCao Are you, by any chance, a smurf account of one of the regulars? Never have I ever seen such a well-defined formation-of-numbers puzzle with the "new contributor" flag on it. Great job, and welcome to PSE!
    $endgroup$
    – Bass
    Feb 28 at 15:31






  • 1




    $begingroup$
    Thank you! I'm not an alt account or anything. As well, I think 6 might be possible but I haven't found a solution. I've only gone down to 7 which I found to be a fun puzzle so I decided to share it.
    $endgroup$
    – Allan Cao
    2 days ago






  • 2




    $begingroup$
    I've got a 5-er too... By thinking out of the box quite a bit... Would be fun to post - but not enough rep...
    $endgroup$
    – ppeterka
    yesterday














  • 10




    $begingroup$
    (1+1+1) / .1 - 1
    $endgroup$
    – shoover
    Feb 28 at 6:28






  • 2




    $begingroup$
    Do you know if there is a solution with less than 7 digits, or are we just hoping there is one?
    $endgroup$
    – luchonacho
    Feb 28 at 11:01






  • 2




    $begingroup$
    @AllanCao Are you, by any chance, a smurf account of one of the regulars? Never have I ever seen such a well-defined formation-of-numbers puzzle with the "new contributor" flag on it. Great job, and welcome to PSE!
    $endgroup$
    – Bass
    Feb 28 at 15:31






  • 1




    $begingroup$
    Thank you! I'm not an alt account or anything. As well, I think 6 might be possible but I haven't found a solution. I've only gone down to 7 which I found to be a fun puzzle so I decided to share it.
    $endgroup$
    – Allan Cao
    2 days ago






  • 2




    $begingroup$
    I've got a 5-er too... By thinking out of the box quite a bit... Would be fun to post - but not enough rep...
    $endgroup$
    – ppeterka
    yesterday








10




10




$begingroup$
(1+1+1) / .1 - 1
$endgroup$
– shoover
Feb 28 at 6:28




$begingroup$
(1+1+1) / .1 - 1
$endgroup$
– shoover
Feb 28 at 6:28




2




2




$begingroup$
Do you know if there is a solution with less than 7 digits, or are we just hoping there is one?
$endgroup$
– luchonacho
Feb 28 at 11:01




$begingroup$
Do you know if there is a solution with less than 7 digits, or are we just hoping there is one?
$endgroup$
– luchonacho
Feb 28 at 11:01




2




2




$begingroup$
@AllanCao Are you, by any chance, a smurf account of one of the regulars? Never have I ever seen such a well-defined formation-of-numbers puzzle with the "new contributor" flag on it. Great job, and welcome to PSE!
$endgroup$
– Bass
Feb 28 at 15:31




$begingroup$
@AllanCao Are you, by any chance, a smurf account of one of the regulars? Never have I ever seen such a well-defined formation-of-numbers puzzle with the "new contributor" flag on it. Great job, and welcome to PSE!
$endgroup$
– Bass
Feb 28 at 15:31




1




1




$begingroup$
Thank you! I'm not an alt account or anything. As well, I think 6 might be possible but I haven't found a solution. I've only gone down to 7 which I found to be a fun puzzle so I decided to share it.
$endgroup$
– Allan Cao
2 days ago




$begingroup$
Thank you! I'm not an alt account or anything. As well, I think 6 might be possible but I haven't found a solution. I've only gone down to 7 which I found to be a fun puzzle so I decided to share it.
$endgroup$
– Allan Cao
2 days ago




2




2




$begingroup$
I've got a 5-er too... By thinking out of the box quite a bit... Would be fun to post - but not enough rep...
$endgroup$
– ppeterka
yesterday




$begingroup$
I've got a 5-er too... By thinking out of the box quite a bit... Would be fun to post - but not enough rep...
$endgroup$
– ppeterka
yesterday










10 Answers
10






active

oldest

votes


















24












$begingroup$

Here's a 7-digit solution:




$(11-1)times(1+1+1)-1$




Searching exhaustively is somewhat complicated. Let's say, broadly, impossible. One is able to take indefinite factorials and raise to ludicrous powers and take arbitrary numbers of square roots. These endeavors would almost certainly lead nowhere but computers don't necessarily know that.



Not only so, but a search that doesn't use symbolic algorithms to handle sqrts and fractions might find something that looks like a solution but is only 29.0000000.



Nonetheless, one can arbitrarily put "reasonable" restrictions on such things and say that we are not going to do, for example, $a^b$ unless $0<a<1000$ and $-20<b<20$.



Doing this, I find that there are 28,948 different numbers that can be reached with six 1s. None of them is 29. My code just tracked one solution for each number that it found and spat out the above solution which suggests to me that it's the only one with 7 digits. I haven't tried to do list them all, but if someone else wants to wade through my code it is below



def factorial(n):
if n<=1: return 1
return n*factorial(n-1)

sols={}
sols[1] = {}
sols[1][1] = "1"
vals = set([1])

digits=2
while digits<=7:
sols[digits] = {}

#concat
concat = "1"*digits
if eval(concat) not in vals:
sols[digits][eval(concat)] = concat
vals.add(eval(concat))

#simple sum
if digits not in vals:
ssum = "1+"*digits
sols[digits][digits] = ssum[:-1]
vals.add(digits)

#partitions
for part1 in range(1,digits):
part2 = digits-part1
if part1>part2: break
for val1 in sols[part1]:
for val2 in sols[part2]:
#multiply
if val1*val2 not in vals:
sols[digits][val1*val2] = "("+sols[part1][val1]+")*("+sols[part2][val2]+")"
vals.add(val1*val2)
#add
if val1+val2 not in vals:
sols[digits][val1+val2] = "("+sols[part1][val1]+")+("+sols[part2][val2]+")"
vals.add(val1+val2)
#divide
if val2 !=0:
if val1/val2 not in vals:
sols[digits][val1/val2] = "("+sols[part1][val1]+")/("+sols[part2][val2]+")"
vals.add(val1/val2)
if val1 !=0:
if val2/val1 not in vals:
sols[digits][val2/val1] = "("+sols[part2][val2]+")/("+sols[part1][val1]+")"
vals.add(val2/val1)
#subtract
if val1-val2 not in vals:
sols[digits][val1-val2] = "("+sols[part1][val1]+")-("+sols[part2][val2]+")"
vals.add(val1-val2)
if val2-val1 not in vals:
sols[digits][val2-val1] = "("+sols[part2][val2]+")-("+sols[part1][val1]+")"
vals.add(val2-val1)
#exponent
if val1 > 0 and val1 < 1000 and abs(val2)<20:
if val1**val2 not in vals:
sols[digits][val1**val2] = "("+sols[part1][val1]+")^("+sols[part2][val2]+")"
vals.add(val1**val2)
if val2 > 0 and val2 < 1000 and abs(val1)<20:
if val2**val1 not in vals:
sols[digits][val2**val1] = "("+sols[part2][val2]+")^("+sols[part1][val1]+")"
vals.add(val2**val1)

#adjustments
k = list(sols[digits].keys())[:]
for val in k:
#Sqrt
if val>0:
if val**0.5 not in vals:
sols[digits][val**0.5] = "sqrt("+sols[digits][val]+")"
vals.add(val**0.5)
if val==int(val) and val<=20:
if factorial(val) not in vals:
sols[digits][factorial(val)] = "("+sols[digits][val]+")!"
vals.add(factorial(val))

if 29 in vals:
break
#next number
print(digits,len(sols[digits]))
digits+=1





share|improve this answer











$endgroup$













  • $begingroup$
    That is the minimum I achieved by referencing the Single Digit Representations of Natural Numbers paper. Hopefully 6 is possible.
    $endgroup$
    – Allan Cao
    Feb 28 at 2:51










  • $begingroup$
    Do you mean that allowing concatenations should reduce it from 7 to 6? Or are the constraints the same in the paper you cite as in the question above?
    $endgroup$
    – Dr Xorile
    Feb 28 at 2:53










  • $begingroup$
    The paper uses different rules.
    $endgroup$
    – Allan Cao
    Feb 28 at 3:04










  • $begingroup$
    I wonder how much CPU power/hour would be needed to catalog all possible 7-digit solutions to this problem
    $endgroup$
    – moonheart08
    2 days ago








  • 2




    $begingroup$
    I'll post my attempt just now if I'm at all successful
    $endgroup$
    – Dr Xorile
    2 days ago



















12












$begingroup$

I thought I had it with this:




$sqrt{((1+1+1)!)!+11^{1+1}}$.




Until I remembered that




there's no function to square a number without using any digits, even though there is a function to take a square root without using any extra digits. For the past hour I've been trying to figure out how to square a number just using the square root.

Since square root is a number raised to the power of $frac{1}{2}$, I'm wondering if I can just take a reciprocal of that for a total of six 1s. My guess is that this isn't allowed because there isn't a way to actually write this down!
$sqrt{((1+1+1)!)!+11^{frac{1}{1/2}}}$







share|improve this answer











$endgroup$













  • $begingroup$
    Curious thing: if $f$ denotes a function (or operation), the inverse operation is commonly denoted by $f^{-1}$ (it is not an exponentiation). So, you might denote as ${sqrt{}}^{-1}$ the inverse operation to square root (i.e. squaring, thus using only one 1). Of course, it is probably not permitted by the rules.
    $endgroup$
    – trolley813
    Feb 28 at 8:57










  • $begingroup$
    Note that $3!! = 3 times 1$ not $(3!)!$ and adding more parentheses makes your first solution a good 7.
    $endgroup$
    – Weather Vane
    Feb 28 at 9:23












  • $begingroup$
    @WeatherVane I didn't realize that double factorial had a different meaning. Of course I meant the iterated factorial. I'll add the parenthesis.
    $endgroup$
    – Amorydai
    Feb 28 at 14:57










  • $begingroup$
    What about negative exponents?
    $endgroup$
    – Canadian Luke
    2 days ago










  • $begingroup$
    I like how multifactorials break every rule of math notation.
    $endgroup$
    – Adonalsium
    yesterday



















12












$begingroup$

As stated by luchonacho, exhaustion will show that it's not possible in less than 7 digits with those operations.



Not showing negative values, as they are symmetric to positive ones, here are the easy attainable integer values:



1 digit:  1
2 digits: 0, 2, 11, 11!, ...
3 digits: 3, 6, 10, 12, 111, 720, 10!, ...
4 digits: 4, 5, 7, 9, 13, 22, 24, 110, 112, 120, 121, 719, 721, 1111, ...
5 digits: 8, 14, 17, 20, 21, 23, 25, 33, 36, 64, 66, 100, 109, 113, 119, 122, 132, 144, 222, 360, ...


And now we know we're stuck, because to achieve 29 (a prime number) in 6 digits, we would actually need any of those values in 5 digits: 28, 30, 784, 900, ... and we don't have them.



As such, any solution for 29 in 7 digits is optimal:




29 = (11−1)×(1+1+1)−1 (found by Dr Xorile)




Note that there may be room for a non-trivial sum of factorial numbers to reach a square power of 29 (or of 29±1), as found by Amorydai, but it's unlikely to beat 7 digits.





For illustration, while all values below 27 are attainable trivially with 6 digits or less, here are some solutions in 7 digits for numbers greater or equal to 28:




28 in 7 digits = (1+1+1)^(1+1+1)+1

28 in 7 digits = (1+1)×(11+1+1+1)

29 in 7 digits = (11−1)×(1+1+1)−1

30 in 6 digits = (11−1)×(1+1+1)

31 in 7 digits = (11−1)×(1+1)+11

31 in 7 digits = (11−1)×(1+1+1)+1

31 in 7 digits = (1+1+1)×11-1-1

32 in 6 digits = (1+1+1)×11-1

33 in 5 digits = (1+1+1)×11

34 in 6 digits = (1+1+1)×11+1

35 in 6 digits = (1+1+1)!^(1+1)-1

36 in 5 digits = (1+1+1)!^(1+1)

37 in 6 digits = (1+1+1)!^(1+1)+1

38 in 7 digits = (1+1+1)!^(1+1)+1+1

39 in 7 digits = (11+1+1)×(1+1+1)

40 in 7 digits = (11-1)×(1+1+1+1)

40 in 7 digits = (11-1)×(1+1)×(1+1)




15 is the lowest positive value requiring 6 digits with this reasoning.
28 is the lowest positive value requiring 7 digits with this reasoning.
41 is the lowest positive value requiring 8 digits with this reasoning.






share|improve this answer











$endgroup$













  • $begingroup$
    Well argued! Can you somehow build a mathematical proof from this line of thinking? It would be great.
    $endgroup$
    – luchonacho
    2 days ago








  • 3




    $begingroup$
    The claim that it's impossible to reach 29 by some iterated factorial, followed by a trivial addition/subtraction, followed by an iterated square root, might be very hard to prove (and necessary, because factorial and square root are free actions in this ruleset).
    $endgroup$
    – Brilliand
    2 days ago










  • $begingroup$
    @Brilliand in x! ± 1, you never have a divisor lower than x, so subsequent square roots will always have a factor greater than x itself.
    $endgroup$
    – Cœur
    2 days ago










  • $begingroup$
    @Cœur could you add two factorials then modify the result? I would hazard a guess that you could find something that roots to a factor smaller than x. Unlikely whether that would be doable in less than 6 though, more as a counterexample to that proof.
    $endgroup$
    – Riker
    2 days ago








  • 1




    $begingroup$
    Note that you can get 27 = (1+1+1)^(1+1+1) for 6 digits. Just for your illustration!
    $endgroup$
    – CriminallyVulgar
    2 days ago



















10












$begingroup$

I managed three 8's:




$(1+1+1+1)!+(1+1+1)!-1=24+6-1$
$(1+1+1)!times((1+1+1)!-1)-1=6times(6-1)-1$
$(11+1+1+1)times(1+1)+1=14times2-1$




and a very dodgy 6:




$11!!!!!!!!-(1+1+1+1)=11times3-4$




and 4:




$(11-1)!!!!!!!-1=10times3-1$







share|improve this answer









$endgroup$









  • 12




    $begingroup$
    ... I don't think that's how factorial works. I'm very sure that is not how factorial works.
    $endgroup$
    – Allan Cao
    Feb 28 at 8:43






  • 9




    $begingroup$
    @AllanCao I believe JonMark Perry is using extended factorial functions. For example, a double factorial is defined to be $n(n-2)(n-4)(n-6)...(3)(1)$ if $n$ is odd, and $n(n-2)(n-4)(n-6)...(4)(2)$ if $n$ is even. If we're calculating a triple-factorial, that evaluates to $n(n-3)(n-6)(n-9)...$. In the case of $(11 - 1)!!!!!!!$ (a seven-factorial), it simplifies like this: $(11 - 1)!!!!!!! = (10)!!!!!!! = (10)(10-7) = (10)(3) = 30$
    $endgroup$
    – Hugh
    Feb 28 at 8:50








  • 2




    $begingroup$
    I see. Since they aren't really factorial (and kind of cheating/cheap), the solution for 6 and 4 don't work.
    $endgroup$
    – Allan Cao
    2 days ago










  • $begingroup$
    multi-factorial, but in most 24-type games they are considered to be a big cheat!!! mind you, so is (1+1).concat(11-1-1). @AllanCao
    $endgroup$
    – JonMark Perry
    2 days ago








  • 2




    $begingroup$
    Which is why the concatenation is only for the original digit.
    $endgroup$
    – Allan Cao
    2 days ago



















8












$begingroup$

According to my analysis, there is no solution using less than 7 digits. The proof would require an exhaustion of every possible operation from 29, reducing the dimensionality of the problem one digit at a time, like a tree. I haven't followed all the leaves of the tree, but it seems clear no one leads to a solution.



To see how my logic works, consider an example:



$29 + 1 = 30$



Here we have a sub problem, which is to find a way to get to 30 using 5 or less digits. You can then follow the leaves of the tree, reducing the dimensionality one at a time, until you see there is no solution. For example, 30 + 1 = 31, and finding 31 from 4 digits, etc. Or 30 + 11 = 41, and so on.



Importantly, many of the branches of the original problem are identical. For example, $29 + 1 = 30$ is the same as $29+1! = 30$; and the subbranch above $30 + 1 = 31$ is exactly the same as $29+(1+1) = 31$ and $29+(1+1)! = 31$.



Same logic applies to, for instance, $29-1=28$. You need to get to 28 using 5 digits. Following the branches seem to lead to nowhere.



Some routes are evidently discarded. For instance, 11! = 39916800, too high. Would need to be scaled with a relatively high number, like 111. But that uses 5 of 6 digits. No way to use the last one meaningfully. Same logic with 111! or other like (11-1)! Other routes, like (1+1+1)!=6, requires you to produce 23 (so 23+6=29) with only 3 digits. Impossible.



No idea why you would ever consider square root as a relevant operation. Temptation here is that somehow a squared root times a natural number ($sqrt{a}b$) will give a natural number. However, this is only true if $ab^2$ is itself the square of a natural number. So you should start by searching for the latter. For example, take 36. It's the square of 6. Can you find natural numbers a and b such that $ab^2=36$? Yes, a=4 and b=3, or a=9 and b=2. But these use too many digits. Follow down the tree and you are done very quickly. What about a=11? It can be seen that it is not possible to find a natural number b such that $sqrt{11b^2}$ is a natural number.



The above applies the same with division, e.g. $frac{sqrt{a}}{b}$ or $frac{b}{sqrt{a}}$ routes.



I haven't given you any proof of my claim and I might be totally wrong. But my methodology (reducing the dimensionality of the problem one at a time) led me to nowhere.






share|improve this answer











$endgroup$









  • 3




    $begingroup$
    My solution for 7 digits uses a square root quite nicely.
    $endgroup$
    – Amorydai
    Feb 28 at 15:15










  • $begingroup$
    @Amorydai True. I did overlook something like $sqrt{a+b}$, focusing instead only on multiplicative forms, like $sqrt{ab}$. I just had another go from derivations of yours, and it seems impossible to reduce it 1 digit. I could be wrong! As I said, I cannot offer proof, only failed trials. I do not want to go down every possible branch of the tree, even though it is possible.
    $endgroup$
    – luchonacho
    2 days ago












  • $begingroup$
    The only cases where $sqrt{a}b$ is a natural with both $a$ and $b$ natural are when $a$ is already a square, or when $b=0$.
    $endgroup$
    – user2357112
    2 days ago






  • 4




    $begingroup$
    Your "reducing the dimensionality one at a time" logic fails to account for the possibility of combining two expressions that both use more than one digit. For example, $(1+1+1)(1+1+1)$ cannot be decomposed into a 5-digit expression and a 1-digit expression.
    $endgroup$
    – user2357112
    2 days ago










  • $begingroup$
    @user2357112 True, but to the extent that these expressions have to be multiplied or added to other terms in order to reach 29, they are covered in my examples. For instance, your example gives a 9. That is precisely one of my examples, where $a=9$ and $b=2$. In case of addition/substraction, you still need to get to 20 (29-9) or 38 (29+9) using the remaining digits.
    $endgroup$
    – luchonacho
    2 days ago





















2












$begingroup$

I can get an answer accurate to within 0.12% with six 1s:




$$sqrt{sqrt{sqrt{11^{11}}}}+1+1 approx 29.0343$$







share|improve this answer









$endgroup$









  • 2




    $begingroup$
    Interesting approach to the question but an exact answer is required.
    $endgroup$
    – Allan Cao
    Feb 28 at 7:25



















2












$begingroup$

Lowest I managed so far is 9 digits:





  • (1 + 1 + 1 + 1)! + 1 + 1 + 1 + 1 + 1;




  • 11*(1 + 1 + 1) - (1 + 1 + 1 + 1)




Some other ways I came up with:





  • (1 + 1)^(1 + 1 + 1 + 1 + 1) - 1 - 1 - 1 (10 digits)




  • (1 + 1 + 1 + 1 + 1)!/(1 + 1 + 1 + 1) - 1 (10 digits)




  • (1 + 1 + 1 + 1 + 1)^(1 + 1) + 1 + 1 + 1 + 1 (11 digits)




  • 11*(1 + 1) + 1 + 1 + 1 + 1 + 1 + 1 + 1 (11 digits)








share|improve this answer











$endgroup$













  • $begingroup$
    Looking at your first 10 and 11 digit solutions, can't you simplify to (1+1+1)^(1+1+1)+1+1 for 8 digits?
    $endgroup$
    – Chronocidal
    Feb 28 at 10:07










  • $begingroup$
    @Chronocidal: Less simplified and more a different solution...
    $endgroup$
    – Chris
    Feb 28 at 14:44



















1












$begingroup$

For the sake of completeness, here's an 8 digit solution:




(1+1+1)^(1+1+1) + 1 + 1







share|improve this answer









$endgroup$





















    1












    $begingroup$

    This was posted before the question was edited to demand an integer, and requires that the rules accept rounding, but it does produce the digits 29. Six is achievable in this way:




    $sqrt{( ( 1 + 1 + 1 )! + 1 )} * 11 approx 29.1033$







    share|improve this answer











    $endgroup$









    • 2




      $begingroup$
      Interesting approach to the question but an exact answer is required.
      $endgroup$
      – Allan Cao
      Feb 28 at 7:25










    • $begingroup$
      You could fix it with the floor: $lfloorsqrt{( ( 1 + 1 + 1 )! + 1 )} * 11rfloor = 29$
      $endgroup$
      – Dr Xorile
      2 days ago












    • $begingroup$
      Floor can't be used. As well, I just assumed that people would take 29 as exactly 29 not around 29.
      $endgroup$
      – Allan Cao
      2 days ago










    • $begingroup$
      Well, as the problem originally stated "make 29" without an explicit restriction to an exact integer result, asking on a puzzling site where people enjoy posting creative/stretch answers will bring one to accept such responses. Before the restriction was imposed, the multiple definitions of "make" might not rule out extra digits appearing.
      $endgroup$
      – a sandwhich
      yesterday





















    0












    $begingroup$


    11+11+11-1-1-1-1=29 This is my best guess.







    share|improve this answer










    New contributor




    Jodi Ansley is a new contributor to this site. Take care in asking for clarification, commenting, and answering.
    Check out our Code of Conduct.






    $endgroup$









    • 7




      $begingroup$
      Even 11 * (1+1+1) is better for the beginning.
      $endgroup$
      – Allan Cao
      Feb 28 at 7:22










    protected by Rubio Feb 28 at 7:46



    Thank you for your interest in this question.
    Because it has attracted low-quality or spam answers that had to be removed, posting an answer now requires 10 reputation on this site (the association bonus does not count).



    Would you like to answer one of these unanswered questions instead?














    10 Answers
    10






    active

    oldest

    votes








    10 Answers
    10






    active

    oldest

    votes









    active

    oldest

    votes






    active

    oldest

    votes









    24












    $begingroup$

    Here's a 7-digit solution:




    $(11-1)times(1+1+1)-1$




    Searching exhaustively is somewhat complicated. Let's say, broadly, impossible. One is able to take indefinite factorials and raise to ludicrous powers and take arbitrary numbers of square roots. These endeavors would almost certainly lead nowhere but computers don't necessarily know that.



    Not only so, but a search that doesn't use symbolic algorithms to handle sqrts and fractions might find something that looks like a solution but is only 29.0000000.



    Nonetheless, one can arbitrarily put "reasonable" restrictions on such things and say that we are not going to do, for example, $a^b$ unless $0<a<1000$ and $-20<b<20$.



    Doing this, I find that there are 28,948 different numbers that can be reached with six 1s. None of them is 29. My code just tracked one solution for each number that it found and spat out the above solution which suggests to me that it's the only one with 7 digits. I haven't tried to do list them all, but if someone else wants to wade through my code it is below



    def factorial(n):
    if n<=1: return 1
    return n*factorial(n-1)

    sols={}
    sols[1] = {}
    sols[1][1] = "1"
    vals = set([1])

    digits=2
    while digits<=7:
    sols[digits] = {}

    #concat
    concat = "1"*digits
    if eval(concat) not in vals:
    sols[digits][eval(concat)] = concat
    vals.add(eval(concat))

    #simple sum
    if digits not in vals:
    ssum = "1+"*digits
    sols[digits][digits] = ssum[:-1]
    vals.add(digits)

    #partitions
    for part1 in range(1,digits):
    part2 = digits-part1
    if part1>part2: break
    for val1 in sols[part1]:
    for val2 in sols[part2]:
    #multiply
    if val1*val2 not in vals:
    sols[digits][val1*val2] = "("+sols[part1][val1]+")*("+sols[part2][val2]+")"
    vals.add(val1*val2)
    #add
    if val1+val2 not in vals:
    sols[digits][val1+val2] = "("+sols[part1][val1]+")+("+sols[part2][val2]+")"
    vals.add(val1+val2)
    #divide
    if val2 !=0:
    if val1/val2 not in vals:
    sols[digits][val1/val2] = "("+sols[part1][val1]+")/("+sols[part2][val2]+")"
    vals.add(val1/val2)
    if val1 !=0:
    if val2/val1 not in vals:
    sols[digits][val2/val1] = "("+sols[part2][val2]+")/("+sols[part1][val1]+")"
    vals.add(val2/val1)
    #subtract
    if val1-val2 not in vals:
    sols[digits][val1-val2] = "("+sols[part1][val1]+")-("+sols[part2][val2]+")"
    vals.add(val1-val2)
    if val2-val1 not in vals:
    sols[digits][val2-val1] = "("+sols[part2][val2]+")-("+sols[part1][val1]+")"
    vals.add(val2-val1)
    #exponent
    if val1 > 0 and val1 < 1000 and abs(val2)<20:
    if val1**val2 not in vals:
    sols[digits][val1**val2] = "("+sols[part1][val1]+")^("+sols[part2][val2]+")"
    vals.add(val1**val2)
    if val2 > 0 and val2 < 1000 and abs(val1)<20:
    if val2**val1 not in vals:
    sols[digits][val2**val1] = "("+sols[part2][val2]+")^("+sols[part1][val1]+")"
    vals.add(val2**val1)

    #adjustments
    k = list(sols[digits].keys())[:]
    for val in k:
    #Sqrt
    if val>0:
    if val**0.5 not in vals:
    sols[digits][val**0.5] = "sqrt("+sols[digits][val]+")"
    vals.add(val**0.5)
    if val==int(val) and val<=20:
    if factorial(val) not in vals:
    sols[digits][factorial(val)] = "("+sols[digits][val]+")!"
    vals.add(factorial(val))

    if 29 in vals:
    break
    #next number
    print(digits,len(sols[digits]))
    digits+=1





    share|improve this answer











    $endgroup$













    • $begingroup$
      That is the minimum I achieved by referencing the Single Digit Representations of Natural Numbers paper. Hopefully 6 is possible.
      $endgroup$
      – Allan Cao
      Feb 28 at 2:51










    • $begingroup$
      Do you mean that allowing concatenations should reduce it from 7 to 6? Or are the constraints the same in the paper you cite as in the question above?
      $endgroup$
      – Dr Xorile
      Feb 28 at 2:53










    • $begingroup$
      The paper uses different rules.
      $endgroup$
      – Allan Cao
      Feb 28 at 3:04










    • $begingroup$
      I wonder how much CPU power/hour would be needed to catalog all possible 7-digit solutions to this problem
      $endgroup$
      – moonheart08
      2 days ago








    • 2




      $begingroup$
      I'll post my attempt just now if I'm at all successful
      $endgroup$
      – Dr Xorile
      2 days ago
















    24












    $begingroup$

    Here's a 7-digit solution:




    $(11-1)times(1+1+1)-1$




    Searching exhaustively is somewhat complicated. Let's say, broadly, impossible. One is able to take indefinite factorials and raise to ludicrous powers and take arbitrary numbers of square roots. These endeavors would almost certainly lead nowhere but computers don't necessarily know that.



    Not only so, but a search that doesn't use symbolic algorithms to handle sqrts and fractions might find something that looks like a solution but is only 29.0000000.



    Nonetheless, one can arbitrarily put "reasonable" restrictions on such things and say that we are not going to do, for example, $a^b$ unless $0<a<1000$ and $-20<b<20$.



    Doing this, I find that there are 28,948 different numbers that can be reached with six 1s. None of them is 29. My code just tracked one solution for each number that it found and spat out the above solution which suggests to me that it's the only one with 7 digits. I haven't tried to do list them all, but if someone else wants to wade through my code it is below



    def factorial(n):
    if n<=1: return 1
    return n*factorial(n-1)

    sols={}
    sols[1] = {}
    sols[1][1] = "1"
    vals = set([1])

    digits=2
    while digits<=7:
    sols[digits] = {}

    #concat
    concat = "1"*digits
    if eval(concat) not in vals:
    sols[digits][eval(concat)] = concat
    vals.add(eval(concat))

    #simple sum
    if digits not in vals:
    ssum = "1+"*digits
    sols[digits][digits] = ssum[:-1]
    vals.add(digits)

    #partitions
    for part1 in range(1,digits):
    part2 = digits-part1
    if part1>part2: break
    for val1 in sols[part1]:
    for val2 in sols[part2]:
    #multiply
    if val1*val2 not in vals:
    sols[digits][val1*val2] = "("+sols[part1][val1]+")*("+sols[part2][val2]+")"
    vals.add(val1*val2)
    #add
    if val1+val2 not in vals:
    sols[digits][val1+val2] = "("+sols[part1][val1]+")+("+sols[part2][val2]+")"
    vals.add(val1+val2)
    #divide
    if val2 !=0:
    if val1/val2 not in vals:
    sols[digits][val1/val2] = "("+sols[part1][val1]+")/("+sols[part2][val2]+")"
    vals.add(val1/val2)
    if val1 !=0:
    if val2/val1 not in vals:
    sols[digits][val2/val1] = "("+sols[part2][val2]+")/("+sols[part1][val1]+")"
    vals.add(val2/val1)
    #subtract
    if val1-val2 not in vals:
    sols[digits][val1-val2] = "("+sols[part1][val1]+")-("+sols[part2][val2]+")"
    vals.add(val1-val2)
    if val2-val1 not in vals:
    sols[digits][val2-val1] = "("+sols[part2][val2]+")-("+sols[part1][val1]+")"
    vals.add(val2-val1)
    #exponent
    if val1 > 0 and val1 < 1000 and abs(val2)<20:
    if val1**val2 not in vals:
    sols[digits][val1**val2] = "("+sols[part1][val1]+")^("+sols[part2][val2]+")"
    vals.add(val1**val2)
    if val2 > 0 and val2 < 1000 and abs(val1)<20:
    if val2**val1 not in vals:
    sols[digits][val2**val1] = "("+sols[part2][val2]+")^("+sols[part1][val1]+")"
    vals.add(val2**val1)

    #adjustments
    k = list(sols[digits].keys())[:]
    for val in k:
    #Sqrt
    if val>0:
    if val**0.5 not in vals:
    sols[digits][val**0.5] = "sqrt("+sols[digits][val]+")"
    vals.add(val**0.5)
    if val==int(val) and val<=20:
    if factorial(val) not in vals:
    sols[digits][factorial(val)] = "("+sols[digits][val]+")!"
    vals.add(factorial(val))

    if 29 in vals:
    break
    #next number
    print(digits,len(sols[digits]))
    digits+=1





    share|improve this answer











    $endgroup$













    • $begingroup$
      That is the minimum I achieved by referencing the Single Digit Representations of Natural Numbers paper. Hopefully 6 is possible.
      $endgroup$
      – Allan Cao
      Feb 28 at 2:51










    • $begingroup$
      Do you mean that allowing concatenations should reduce it from 7 to 6? Or are the constraints the same in the paper you cite as in the question above?
      $endgroup$
      – Dr Xorile
      Feb 28 at 2:53










    • $begingroup$
      The paper uses different rules.
      $endgroup$
      – Allan Cao
      Feb 28 at 3:04










    • $begingroup$
      I wonder how much CPU power/hour would be needed to catalog all possible 7-digit solutions to this problem
      $endgroup$
      – moonheart08
      2 days ago








    • 2




      $begingroup$
      I'll post my attempt just now if I'm at all successful
      $endgroup$
      – Dr Xorile
      2 days ago














    24












    24








    24





    $begingroup$

    Here's a 7-digit solution:




    $(11-1)times(1+1+1)-1$




    Searching exhaustively is somewhat complicated. Let's say, broadly, impossible. One is able to take indefinite factorials and raise to ludicrous powers and take arbitrary numbers of square roots. These endeavors would almost certainly lead nowhere but computers don't necessarily know that.



    Not only so, but a search that doesn't use symbolic algorithms to handle sqrts and fractions might find something that looks like a solution but is only 29.0000000.



    Nonetheless, one can arbitrarily put "reasonable" restrictions on such things and say that we are not going to do, for example, $a^b$ unless $0<a<1000$ and $-20<b<20$.



    Doing this, I find that there are 28,948 different numbers that can be reached with six 1s. None of them is 29. My code just tracked one solution for each number that it found and spat out the above solution which suggests to me that it's the only one with 7 digits. I haven't tried to do list them all, but if someone else wants to wade through my code it is below



    def factorial(n):
    if n<=1: return 1
    return n*factorial(n-1)

    sols={}
    sols[1] = {}
    sols[1][1] = "1"
    vals = set([1])

    digits=2
    while digits<=7:
    sols[digits] = {}

    #concat
    concat = "1"*digits
    if eval(concat) not in vals:
    sols[digits][eval(concat)] = concat
    vals.add(eval(concat))

    #simple sum
    if digits not in vals:
    ssum = "1+"*digits
    sols[digits][digits] = ssum[:-1]
    vals.add(digits)

    #partitions
    for part1 in range(1,digits):
    part2 = digits-part1
    if part1>part2: break
    for val1 in sols[part1]:
    for val2 in sols[part2]:
    #multiply
    if val1*val2 not in vals:
    sols[digits][val1*val2] = "("+sols[part1][val1]+")*("+sols[part2][val2]+")"
    vals.add(val1*val2)
    #add
    if val1+val2 not in vals:
    sols[digits][val1+val2] = "("+sols[part1][val1]+")+("+sols[part2][val2]+")"
    vals.add(val1+val2)
    #divide
    if val2 !=0:
    if val1/val2 not in vals:
    sols[digits][val1/val2] = "("+sols[part1][val1]+")/("+sols[part2][val2]+")"
    vals.add(val1/val2)
    if val1 !=0:
    if val2/val1 not in vals:
    sols[digits][val2/val1] = "("+sols[part2][val2]+")/("+sols[part1][val1]+")"
    vals.add(val2/val1)
    #subtract
    if val1-val2 not in vals:
    sols[digits][val1-val2] = "("+sols[part1][val1]+")-("+sols[part2][val2]+")"
    vals.add(val1-val2)
    if val2-val1 not in vals:
    sols[digits][val2-val1] = "("+sols[part2][val2]+")-("+sols[part1][val1]+")"
    vals.add(val2-val1)
    #exponent
    if val1 > 0 and val1 < 1000 and abs(val2)<20:
    if val1**val2 not in vals:
    sols[digits][val1**val2] = "("+sols[part1][val1]+")^("+sols[part2][val2]+")"
    vals.add(val1**val2)
    if val2 > 0 and val2 < 1000 and abs(val1)<20:
    if val2**val1 not in vals:
    sols[digits][val2**val1] = "("+sols[part2][val2]+")^("+sols[part1][val1]+")"
    vals.add(val2**val1)

    #adjustments
    k = list(sols[digits].keys())[:]
    for val in k:
    #Sqrt
    if val>0:
    if val**0.5 not in vals:
    sols[digits][val**0.5] = "sqrt("+sols[digits][val]+")"
    vals.add(val**0.5)
    if val==int(val) and val<=20:
    if factorial(val) not in vals:
    sols[digits][factorial(val)] = "("+sols[digits][val]+")!"
    vals.add(factorial(val))

    if 29 in vals:
    break
    #next number
    print(digits,len(sols[digits]))
    digits+=1





    share|improve this answer











    $endgroup$



    Here's a 7-digit solution:




    $(11-1)times(1+1+1)-1$




    Searching exhaustively is somewhat complicated. Let's say, broadly, impossible. One is able to take indefinite factorials and raise to ludicrous powers and take arbitrary numbers of square roots. These endeavors would almost certainly lead nowhere but computers don't necessarily know that.



    Not only so, but a search that doesn't use symbolic algorithms to handle sqrts and fractions might find something that looks like a solution but is only 29.0000000.



    Nonetheless, one can arbitrarily put "reasonable" restrictions on such things and say that we are not going to do, for example, $a^b$ unless $0<a<1000$ and $-20<b<20$.



    Doing this, I find that there are 28,948 different numbers that can be reached with six 1s. None of them is 29. My code just tracked one solution for each number that it found and spat out the above solution which suggests to me that it's the only one with 7 digits. I haven't tried to do list them all, but if someone else wants to wade through my code it is below



    def factorial(n):
    if n<=1: return 1
    return n*factorial(n-1)

    sols={}
    sols[1] = {}
    sols[1][1] = "1"
    vals = set([1])

    digits=2
    while digits<=7:
    sols[digits] = {}

    #concat
    concat = "1"*digits
    if eval(concat) not in vals:
    sols[digits][eval(concat)] = concat
    vals.add(eval(concat))

    #simple sum
    if digits not in vals:
    ssum = "1+"*digits
    sols[digits][digits] = ssum[:-1]
    vals.add(digits)

    #partitions
    for part1 in range(1,digits):
    part2 = digits-part1
    if part1>part2: break
    for val1 in sols[part1]:
    for val2 in sols[part2]:
    #multiply
    if val1*val2 not in vals:
    sols[digits][val1*val2] = "("+sols[part1][val1]+")*("+sols[part2][val2]+")"
    vals.add(val1*val2)
    #add
    if val1+val2 not in vals:
    sols[digits][val1+val2] = "("+sols[part1][val1]+")+("+sols[part2][val2]+")"
    vals.add(val1+val2)
    #divide
    if val2 !=0:
    if val1/val2 not in vals:
    sols[digits][val1/val2] = "("+sols[part1][val1]+")/("+sols[part2][val2]+")"
    vals.add(val1/val2)
    if val1 !=0:
    if val2/val1 not in vals:
    sols[digits][val2/val1] = "("+sols[part2][val2]+")/("+sols[part1][val1]+")"
    vals.add(val2/val1)
    #subtract
    if val1-val2 not in vals:
    sols[digits][val1-val2] = "("+sols[part1][val1]+")-("+sols[part2][val2]+")"
    vals.add(val1-val2)
    if val2-val1 not in vals:
    sols[digits][val2-val1] = "("+sols[part2][val2]+")-("+sols[part1][val1]+")"
    vals.add(val2-val1)
    #exponent
    if val1 > 0 and val1 < 1000 and abs(val2)<20:
    if val1**val2 not in vals:
    sols[digits][val1**val2] = "("+sols[part1][val1]+")^("+sols[part2][val2]+")"
    vals.add(val1**val2)
    if val2 > 0 and val2 < 1000 and abs(val1)<20:
    if val2**val1 not in vals:
    sols[digits][val2**val1] = "("+sols[part2][val2]+")^("+sols[part1][val1]+")"
    vals.add(val2**val1)

    #adjustments
    k = list(sols[digits].keys())[:]
    for val in k:
    #Sqrt
    if val>0:
    if val**0.5 not in vals:
    sols[digits][val**0.5] = "sqrt("+sols[digits][val]+")"
    vals.add(val**0.5)
    if val==int(val) and val<=20:
    if factorial(val) not in vals:
    sols[digits][factorial(val)] = "("+sols[digits][val]+")!"
    vals.add(factorial(val))

    if 29 in vals:
    break
    #next number
    print(digits,len(sols[digits]))
    digits+=1






    share|improve this answer














    share|improve this answer



    share|improve this answer








    edited 2 days ago

























    answered Feb 28 at 2:40









    Dr XorileDr Xorile

    13.2k22570




    13.2k22570












    • $begingroup$
      That is the minimum I achieved by referencing the Single Digit Representations of Natural Numbers paper. Hopefully 6 is possible.
      $endgroup$
      – Allan Cao
      Feb 28 at 2:51










    • $begingroup$
      Do you mean that allowing concatenations should reduce it from 7 to 6? Or are the constraints the same in the paper you cite as in the question above?
      $endgroup$
      – Dr Xorile
      Feb 28 at 2:53










    • $begingroup$
      The paper uses different rules.
      $endgroup$
      – Allan Cao
      Feb 28 at 3:04










    • $begingroup$
      I wonder how much CPU power/hour would be needed to catalog all possible 7-digit solutions to this problem
      $endgroup$
      – moonheart08
      2 days ago








    • 2




      $begingroup$
      I'll post my attempt just now if I'm at all successful
      $endgroup$
      – Dr Xorile
      2 days ago


















    • $begingroup$
      That is the minimum I achieved by referencing the Single Digit Representations of Natural Numbers paper. Hopefully 6 is possible.
      $endgroup$
      – Allan Cao
      Feb 28 at 2:51










    • $begingroup$
      Do you mean that allowing concatenations should reduce it from 7 to 6? Or are the constraints the same in the paper you cite as in the question above?
      $endgroup$
      – Dr Xorile
      Feb 28 at 2:53










    • $begingroup$
      The paper uses different rules.
      $endgroup$
      – Allan Cao
      Feb 28 at 3:04










    • $begingroup$
      I wonder how much CPU power/hour would be needed to catalog all possible 7-digit solutions to this problem
      $endgroup$
      – moonheart08
      2 days ago








    • 2




      $begingroup$
      I'll post my attempt just now if I'm at all successful
      $endgroup$
      – Dr Xorile
      2 days ago
















    $begingroup$
    That is the minimum I achieved by referencing the Single Digit Representations of Natural Numbers paper. Hopefully 6 is possible.
    $endgroup$
    – Allan Cao
    Feb 28 at 2:51




    $begingroup$
    That is the minimum I achieved by referencing the Single Digit Representations of Natural Numbers paper. Hopefully 6 is possible.
    $endgroup$
    – Allan Cao
    Feb 28 at 2:51












    $begingroup$
    Do you mean that allowing concatenations should reduce it from 7 to 6? Or are the constraints the same in the paper you cite as in the question above?
    $endgroup$
    – Dr Xorile
    Feb 28 at 2:53




    $begingroup$
    Do you mean that allowing concatenations should reduce it from 7 to 6? Or are the constraints the same in the paper you cite as in the question above?
    $endgroup$
    – Dr Xorile
    Feb 28 at 2:53












    $begingroup$
    The paper uses different rules.
    $endgroup$
    – Allan Cao
    Feb 28 at 3:04




    $begingroup$
    The paper uses different rules.
    $endgroup$
    – Allan Cao
    Feb 28 at 3:04












    $begingroup$
    I wonder how much CPU power/hour would be needed to catalog all possible 7-digit solutions to this problem
    $endgroup$
    – moonheart08
    2 days ago






    $begingroup$
    I wonder how much CPU power/hour would be needed to catalog all possible 7-digit solutions to this problem
    $endgroup$
    – moonheart08
    2 days ago






    2




    2




    $begingroup$
    I'll post my attempt just now if I'm at all successful
    $endgroup$
    – Dr Xorile
    2 days ago




    $begingroup$
    I'll post my attempt just now if I'm at all successful
    $endgroup$
    – Dr Xorile
    2 days ago











    12












    $begingroup$

    I thought I had it with this:




    $sqrt{((1+1+1)!)!+11^{1+1}}$.




    Until I remembered that




    there's no function to square a number without using any digits, even though there is a function to take a square root without using any extra digits. For the past hour I've been trying to figure out how to square a number just using the square root.

    Since square root is a number raised to the power of $frac{1}{2}$, I'm wondering if I can just take a reciprocal of that for a total of six 1s. My guess is that this isn't allowed because there isn't a way to actually write this down!
    $sqrt{((1+1+1)!)!+11^{frac{1}{1/2}}}$







    share|improve this answer











    $endgroup$













    • $begingroup$
      Curious thing: if $f$ denotes a function (or operation), the inverse operation is commonly denoted by $f^{-1}$ (it is not an exponentiation). So, you might denote as ${sqrt{}}^{-1}$ the inverse operation to square root (i.e. squaring, thus using only one 1). Of course, it is probably not permitted by the rules.
      $endgroup$
      – trolley813
      Feb 28 at 8:57










    • $begingroup$
      Note that $3!! = 3 times 1$ not $(3!)!$ and adding more parentheses makes your first solution a good 7.
      $endgroup$
      – Weather Vane
      Feb 28 at 9:23












    • $begingroup$
      @WeatherVane I didn't realize that double factorial had a different meaning. Of course I meant the iterated factorial. I'll add the parenthesis.
      $endgroup$
      – Amorydai
      Feb 28 at 14:57










    • $begingroup$
      What about negative exponents?
      $endgroup$
      – Canadian Luke
      2 days ago










    • $begingroup$
      I like how multifactorials break every rule of math notation.
      $endgroup$
      – Adonalsium
      yesterday
















    12












    $begingroup$

    I thought I had it with this:




    $sqrt{((1+1+1)!)!+11^{1+1}}$.




    Until I remembered that




    there's no function to square a number without using any digits, even though there is a function to take a square root without using any extra digits. For the past hour I've been trying to figure out how to square a number just using the square root.

    Since square root is a number raised to the power of $frac{1}{2}$, I'm wondering if I can just take a reciprocal of that for a total of six 1s. My guess is that this isn't allowed because there isn't a way to actually write this down!
    $sqrt{((1+1+1)!)!+11^{frac{1}{1/2}}}$







    share|improve this answer











    $endgroup$













    • $begingroup$
      Curious thing: if $f$ denotes a function (or operation), the inverse operation is commonly denoted by $f^{-1}$ (it is not an exponentiation). So, you might denote as ${sqrt{}}^{-1}$ the inverse operation to square root (i.e. squaring, thus using only one 1). Of course, it is probably not permitted by the rules.
      $endgroup$
      – trolley813
      Feb 28 at 8:57










    • $begingroup$
      Note that $3!! = 3 times 1$ not $(3!)!$ and adding more parentheses makes your first solution a good 7.
      $endgroup$
      – Weather Vane
      Feb 28 at 9:23












    • $begingroup$
      @WeatherVane I didn't realize that double factorial had a different meaning. Of course I meant the iterated factorial. I'll add the parenthesis.
      $endgroup$
      – Amorydai
      Feb 28 at 14:57










    • $begingroup$
      What about negative exponents?
      $endgroup$
      – Canadian Luke
      2 days ago










    • $begingroup$
      I like how multifactorials break every rule of math notation.
      $endgroup$
      – Adonalsium
      yesterday














    12












    12








    12





    $begingroup$

    I thought I had it with this:




    $sqrt{((1+1+1)!)!+11^{1+1}}$.




    Until I remembered that




    there's no function to square a number without using any digits, even though there is a function to take a square root without using any extra digits. For the past hour I've been trying to figure out how to square a number just using the square root.

    Since square root is a number raised to the power of $frac{1}{2}$, I'm wondering if I can just take a reciprocal of that for a total of six 1s. My guess is that this isn't allowed because there isn't a way to actually write this down!
    $sqrt{((1+1+1)!)!+11^{frac{1}{1/2}}}$







    share|improve this answer











    $endgroup$



    I thought I had it with this:




    $sqrt{((1+1+1)!)!+11^{1+1}}$.




    Until I remembered that




    there's no function to square a number without using any digits, even though there is a function to take a square root without using any extra digits. For the past hour I've been trying to figure out how to square a number just using the square root.

    Since square root is a number raised to the power of $frac{1}{2}$, I'm wondering if I can just take a reciprocal of that for a total of six 1s. My guess is that this isn't allowed because there isn't a way to actually write this down!
    $sqrt{((1+1+1)!)!+11^{frac{1}{1/2}}}$








    share|improve this answer














    share|improve this answer



    share|improve this answer








    edited Feb 28 at 14:58

























    answered Feb 28 at 3:31









    AmorydaiAmorydai

    65610




    65610












    • $begingroup$
      Curious thing: if $f$ denotes a function (or operation), the inverse operation is commonly denoted by $f^{-1}$ (it is not an exponentiation). So, you might denote as ${sqrt{}}^{-1}$ the inverse operation to square root (i.e. squaring, thus using only one 1). Of course, it is probably not permitted by the rules.
      $endgroup$
      – trolley813
      Feb 28 at 8:57










    • $begingroup$
      Note that $3!! = 3 times 1$ not $(3!)!$ and adding more parentheses makes your first solution a good 7.
      $endgroup$
      – Weather Vane
      Feb 28 at 9:23












    • $begingroup$
      @WeatherVane I didn't realize that double factorial had a different meaning. Of course I meant the iterated factorial. I'll add the parenthesis.
      $endgroup$
      – Amorydai
      Feb 28 at 14:57










    • $begingroup$
      What about negative exponents?
      $endgroup$
      – Canadian Luke
      2 days ago










    • $begingroup$
      I like how multifactorials break every rule of math notation.
      $endgroup$
      – Adonalsium
      yesterday


















    • $begingroup$
      Curious thing: if $f$ denotes a function (or operation), the inverse operation is commonly denoted by $f^{-1}$ (it is not an exponentiation). So, you might denote as ${sqrt{}}^{-1}$ the inverse operation to square root (i.e. squaring, thus using only one 1). Of course, it is probably not permitted by the rules.
      $endgroup$
      – trolley813
      Feb 28 at 8:57










    • $begingroup$
      Note that $3!! = 3 times 1$ not $(3!)!$ and adding more parentheses makes your first solution a good 7.
      $endgroup$
      – Weather Vane
      Feb 28 at 9:23












    • $begingroup$
      @WeatherVane I didn't realize that double factorial had a different meaning. Of course I meant the iterated factorial. I'll add the parenthesis.
      $endgroup$
      – Amorydai
      Feb 28 at 14:57










    • $begingroup$
      What about negative exponents?
      $endgroup$
      – Canadian Luke
      2 days ago










    • $begingroup$
      I like how multifactorials break every rule of math notation.
      $endgroup$
      – Adonalsium
      yesterday
















    $begingroup$
    Curious thing: if $f$ denotes a function (or operation), the inverse operation is commonly denoted by $f^{-1}$ (it is not an exponentiation). So, you might denote as ${sqrt{}}^{-1}$ the inverse operation to square root (i.e. squaring, thus using only one 1). Of course, it is probably not permitted by the rules.
    $endgroup$
    – trolley813
    Feb 28 at 8:57




    $begingroup$
    Curious thing: if $f$ denotes a function (or operation), the inverse operation is commonly denoted by $f^{-1}$ (it is not an exponentiation). So, you might denote as ${sqrt{}}^{-1}$ the inverse operation to square root (i.e. squaring, thus using only one 1). Of course, it is probably not permitted by the rules.
    $endgroup$
    – trolley813
    Feb 28 at 8:57












    $begingroup$
    Note that $3!! = 3 times 1$ not $(3!)!$ and adding more parentheses makes your first solution a good 7.
    $endgroup$
    – Weather Vane
    Feb 28 at 9:23






    $begingroup$
    Note that $3!! = 3 times 1$ not $(3!)!$ and adding more parentheses makes your first solution a good 7.
    $endgroup$
    – Weather Vane
    Feb 28 at 9:23














    $begingroup$
    @WeatherVane I didn't realize that double factorial had a different meaning. Of course I meant the iterated factorial. I'll add the parenthesis.
    $endgroup$
    – Amorydai
    Feb 28 at 14:57




    $begingroup$
    @WeatherVane I didn't realize that double factorial had a different meaning. Of course I meant the iterated factorial. I'll add the parenthesis.
    $endgroup$
    – Amorydai
    Feb 28 at 14:57












    $begingroup$
    What about negative exponents?
    $endgroup$
    – Canadian Luke
    2 days ago




    $begingroup$
    What about negative exponents?
    $endgroup$
    – Canadian Luke
    2 days ago












    $begingroup$
    I like how multifactorials break every rule of math notation.
    $endgroup$
    – Adonalsium
    yesterday




    $begingroup$
    I like how multifactorials break every rule of math notation.
    $endgroup$
    – Adonalsium
    yesterday











    12












    $begingroup$

    As stated by luchonacho, exhaustion will show that it's not possible in less than 7 digits with those operations.



    Not showing negative values, as they are symmetric to positive ones, here are the easy attainable integer values:



    1 digit:  1
    2 digits: 0, 2, 11, 11!, ...
    3 digits: 3, 6, 10, 12, 111, 720, 10!, ...
    4 digits: 4, 5, 7, 9, 13, 22, 24, 110, 112, 120, 121, 719, 721, 1111, ...
    5 digits: 8, 14, 17, 20, 21, 23, 25, 33, 36, 64, 66, 100, 109, 113, 119, 122, 132, 144, 222, 360, ...


    And now we know we're stuck, because to achieve 29 (a prime number) in 6 digits, we would actually need any of those values in 5 digits: 28, 30, 784, 900, ... and we don't have them.



    As such, any solution for 29 in 7 digits is optimal:




    29 = (11−1)×(1+1+1)−1 (found by Dr Xorile)




    Note that there may be room for a non-trivial sum of factorial numbers to reach a square power of 29 (or of 29±1), as found by Amorydai, but it's unlikely to beat 7 digits.





    For illustration, while all values below 27 are attainable trivially with 6 digits or less, here are some solutions in 7 digits for numbers greater or equal to 28:




    28 in 7 digits = (1+1+1)^(1+1+1)+1

    28 in 7 digits = (1+1)×(11+1+1+1)

    29 in 7 digits = (11−1)×(1+1+1)−1

    30 in 6 digits = (11−1)×(1+1+1)

    31 in 7 digits = (11−1)×(1+1)+11

    31 in 7 digits = (11−1)×(1+1+1)+1

    31 in 7 digits = (1+1+1)×11-1-1

    32 in 6 digits = (1+1+1)×11-1

    33 in 5 digits = (1+1+1)×11

    34 in 6 digits = (1+1+1)×11+1

    35 in 6 digits = (1+1+1)!^(1+1)-1

    36 in 5 digits = (1+1+1)!^(1+1)

    37 in 6 digits = (1+1+1)!^(1+1)+1

    38 in 7 digits = (1+1+1)!^(1+1)+1+1

    39 in 7 digits = (11+1+1)×(1+1+1)

    40 in 7 digits = (11-1)×(1+1+1+1)

    40 in 7 digits = (11-1)×(1+1)×(1+1)




    15 is the lowest positive value requiring 6 digits with this reasoning.
    28 is the lowest positive value requiring 7 digits with this reasoning.
    41 is the lowest positive value requiring 8 digits with this reasoning.






    share|improve this answer











    $endgroup$













    • $begingroup$
      Well argued! Can you somehow build a mathematical proof from this line of thinking? It would be great.
      $endgroup$
      – luchonacho
      2 days ago








    • 3




      $begingroup$
      The claim that it's impossible to reach 29 by some iterated factorial, followed by a trivial addition/subtraction, followed by an iterated square root, might be very hard to prove (and necessary, because factorial and square root are free actions in this ruleset).
      $endgroup$
      – Brilliand
      2 days ago










    • $begingroup$
      @Brilliand in x! ± 1, you never have a divisor lower than x, so subsequent square roots will always have a factor greater than x itself.
      $endgroup$
      – Cœur
      2 days ago










    • $begingroup$
      @Cœur could you add two factorials then modify the result? I would hazard a guess that you could find something that roots to a factor smaller than x. Unlikely whether that would be doable in less than 6 though, more as a counterexample to that proof.
      $endgroup$
      – Riker
      2 days ago








    • 1




      $begingroup$
      Note that you can get 27 = (1+1+1)^(1+1+1) for 6 digits. Just for your illustration!
      $endgroup$
      – CriminallyVulgar
      2 days ago
















    12












    $begingroup$

    As stated by luchonacho, exhaustion will show that it's not possible in less than 7 digits with those operations.



    Not showing negative values, as they are symmetric to positive ones, here are the easy attainable integer values:



    1 digit:  1
    2 digits: 0, 2, 11, 11!, ...
    3 digits: 3, 6, 10, 12, 111, 720, 10!, ...
    4 digits: 4, 5, 7, 9, 13, 22, 24, 110, 112, 120, 121, 719, 721, 1111, ...
    5 digits: 8, 14, 17, 20, 21, 23, 25, 33, 36, 64, 66, 100, 109, 113, 119, 122, 132, 144, 222, 360, ...


    And now we know we're stuck, because to achieve 29 (a prime number) in 6 digits, we would actually need any of those values in 5 digits: 28, 30, 784, 900, ... and we don't have them.



    As such, any solution for 29 in 7 digits is optimal:




    29 = (11−1)×(1+1+1)−1 (found by Dr Xorile)




    Note that there may be room for a non-trivial sum of factorial numbers to reach a square power of 29 (or of 29±1), as found by Amorydai, but it's unlikely to beat 7 digits.





    For illustration, while all values below 27 are attainable trivially with 6 digits or less, here are some solutions in 7 digits for numbers greater or equal to 28:




    28 in 7 digits = (1+1+1)^(1+1+1)+1

    28 in 7 digits = (1+1)×(11+1+1+1)

    29 in 7 digits = (11−1)×(1+1+1)−1

    30 in 6 digits = (11−1)×(1+1+1)

    31 in 7 digits = (11−1)×(1+1)+11

    31 in 7 digits = (11−1)×(1+1+1)+1

    31 in 7 digits = (1+1+1)×11-1-1

    32 in 6 digits = (1+1+1)×11-1

    33 in 5 digits = (1+1+1)×11

    34 in 6 digits = (1+1+1)×11+1

    35 in 6 digits = (1+1+1)!^(1+1)-1

    36 in 5 digits = (1+1+1)!^(1+1)

    37 in 6 digits = (1+1+1)!^(1+1)+1

    38 in 7 digits = (1+1+1)!^(1+1)+1+1

    39 in 7 digits = (11+1+1)×(1+1+1)

    40 in 7 digits = (11-1)×(1+1+1+1)

    40 in 7 digits = (11-1)×(1+1)×(1+1)




    15 is the lowest positive value requiring 6 digits with this reasoning.
    28 is the lowest positive value requiring 7 digits with this reasoning.
    41 is the lowest positive value requiring 8 digits with this reasoning.






    share|improve this answer











    $endgroup$













    • $begingroup$
      Well argued! Can you somehow build a mathematical proof from this line of thinking? It would be great.
      $endgroup$
      – luchonacho
      2 days ago








    • 3




      $begingroup$
      The claim that it's impossible to reach 29 by some iterated factorial, followed by a trivial addition/subtraction, followed by an iterated square root, might be very hard to prove (and necessary, because factorial and square root are free actions in this ruleset).
      $endgroup$
      – Brilliand
      2 days ago










    • $begingroup$
      @Brilliand in x! ± 1, you never have a divisor lower than x, so subsequent square roots will always have a factor greater than x itself.
      $endgroup$
      – Cœur
      2 days ago










    • $begingroup$
      @Cœur could you add two factorials then modify the result? I would hazard a guess that you could find something that roots to a factor smaller than x. Unlikely whether that would be doable in less than 6 though, more as a counterexample to that proof.
      $endgroup$
      – Riker
      2 days ago








    • 1




      $begingroup$
      Note that you can get 27 = (1+1+1)^(1+1+1) for 6 digits. Just for your illustration!
      $endgroup$
      – CriminallyVulgar
      2 days ago














    12












    12








    12





    $begingroup$

    As stated by luchonacho, exhaustion will show that it's not possible in less than 7 digits with those operations.



    Not showing negative values, as they are symmetric to positive ones, here are the easy attainable integer values:



    1 digit:  1
    2 digits: 0, 2, 11, 11!, ...
    3 digits: 3, 6, 10, 12, 111, 720, 10!, ...
    4 digits: 4, 5, 7, 9, 13, 22, 24, 110, 112, 120, 121, 719, 721, 1111, ...
    5 digits: 8, 14, 17, 20, 21, 23, 25, 33, 36, 64, 66, 100, 109, 113, 119, 122, 132, 144, 222, 360, ...


    And now we know we're stuck, because to achieve 29 (a prime number) in 6 digits, we would actually need any of those values in 5 digits: 28, 30, 784, 900, ... and we don't have them.



    As such, any solution for 29 in 7 digits is optimal:




    29 = (11−1)×(1+1+1)−1 (found by Dr Xorile)




    Note that there may be room for a non-trivial sum of factorial numbers to reach a square power of 29 (or of 29±1), as found by Amorydai, but it's unlikely to beat 7 digits.





    For illustration, while all values below 27 are attainable trivially with 6 digits or less, here are some solutions in 7 digits for numbers greater or equal to 28:




    28 in 7 digits = (1+1+1)^(1+1+1)+1

    28 in 7 digits = (1+1)×(11+1+1+1)

    29 in 7 digits = (11−1)×(1+1+1)−1

    30 in 6 digits = (11−1)×(1+1+1)

    31 in 7 digits = (11−1)×(1+1)+11

    31 in 7 digits = (11−1)×(1+1+1)+1

    31 in 7 digits = (1+1+1)×11-1-1

    32 in 6 digits = (1+1+1)×11-1

    33 in 5 digits = (1+1+1)×11

    34 in 6 digits = (1+1+1)×11+1

    35 in 6 digits = (1+1+1)!^(1+1)-1

    36 in 5 digits = (1+1+1)!^(1+1)

    37 in 6 digits = (1+1+1)!^(1+1)+1

    38 in 7 digits = (1+1+1)!^(1+1)+1+1

    39 in 7 digits = (11+1+1)×(1+1+1)

    40 in 7 digits = (11-1)×(1+1+1+1)

    40 in 7 digits = (11-1)×(1+1)×(1+1)




    15 is the lowest positive value requiring 6 digits with this reasoning.
    28 is the lowest positive value requiring 7 digits with this reasoning.
    41 is the lowest positive value requiring 8 digits with this reasoning.






    share|improve this answer











    $endgroup$



    As stated by luchonacho, exhaustion will show that it's not possible in less than 7 digits with those operations.



    Not showing negative values, as they are symmetric to positive ones, here are the easy attainable integer values:



    1 digit:  1
    2 digits: 0, 2, 11, 11!, ...
    3 digits: 3, 6, 10, 12, 111, 720, 10!, ...
    4 digits: 4, 5, 7, 9, 13, 22, 24, 110, 112, 120, 121, 719, 721, 1111, ...
    5 digits: 8, 14, 17, 20, 21, 23, 25, 33, 36, 64, 66, 100, 109, 113, 119, 122, 132, 144, 222, 360, ...


    And now we know we're stuck, because to achieve 29 (a prime number) in 6 digits, we would actually need any of those values in 5 digits: 28, 30, 784, 900, ... and we don't have them.



    As such, any solution for 29 in 7 digits is optimal:




    29 = (11−1)×(1+1+1)−1 (found by Dr Xorile)




    Note that there may be room for a non-trivial sum of factorial numbers to reach a square power of 29 (or of 29±1), as found by Amorydai, but it's unlikely to beat 7 digits.





    For illustration, while all values below 27 are attainable trivially with 6 digits or less, here are some solutions in 7 digits for numbers greater or equal to 28:




    28 in 7 digits = (1+1+1)^(1+1+1)+1

    28 in 7 digits = (1+1)×(11+1+1+1)

    29 in 7 digits = (11−1)×(1+1+1)−1

    30 in 6 digits = (11−1)×(1+1+1)

    31 in 7 digits = (11−1)×(1+1)+11

    31 in 7 digits = (11−1)×(1+1+1)+1

    31 in 7 digits = (1+1+1)×11-1-1

    32 in 6 digits = (1+1+1)×11-1

    33 in 5 digits = (1+1+1)×11

    34 in 6 digits = (1+1+1)×11+1

    35 in 6 digits = (1+1+1)!^(1+1)-1

    36 in 5 digits = (1+1+1)!^(1+1)

    37 in 6 digits = (1+1+1)!^(1+1)+1

    38 in 7 digits = (1+1+1)!^(1+1)+1+1

    39 in 7 digits = (11+1+1)×(1+1+1)

    40 in 7 digits = (11-1)×(1+1+1+1)

    40 in 7 digits = (11-1)×(1+1)×(1+1)




    15 is the lowest positive value requiring 6 digits with this reasoning.
    28 is the lowest positive value requiring 7 digits with this reasoning.
    41 is the lowest positive value requiring 8 digits with this reasoning.







    share|improve this answer














    share|improve this answer



    share|improve this answer








    edited yesterday

























    answered Feb 28 at 14:26









    CœurCœur

    24117




    24117












    • $begingroup$
      Well argued! Can you somehow build a mathematical proof from this line of thinking? It would be great.
      $endgroup$
      – luchonacho
      2 days ago








    • 3




      $begingroup$
      The claim that it's impossible to reach 29 by some iterated factorial, followed by a trivial addition/subtraction, followed by an iterated square root, might be very hard to prove (and necessary, because factorial and square root are free actions in this ruleset).
      $endgroup$
      – Brilliand
      2 days ago










    • $begingroup$
      @Brilliand in x! ± 1, you never have a divisor lower than x, so subsequent square roots will always have a factor greater than x itself.
      $endgroup$
      – Cœur
      2 days ago










    • $begingroup$
      @Cœur could you add two factorials then modify the result? I would hazard a guess that you could find something that roots to a factor smaller than x. Unlikely whether that would be doable in less than 6 though, more as a counterexample to that proof.
      $endgroup$
      – Riker
      2 days ago








    • 1




      $begingroup$
      Note that you can get 27 = (1+1+1)^(1+1+1) for 6 digits. Just for your illustration!
      $endgroup$
      – CriminallyVulgar
      2 days ago


















    • $begingroup$
      Well argued! Can you somehow build a mathematical proof from this line of thinking? It would be great.
      $endgroup$
      – luchonacho
      2 days ago








    • 3




      $begingroup$
      The claim that it's impossible to reach 29 by some iterated factorial, followed by a trivial addition/subtraction, followed by an iterated square root, might be very hard to prove (and necessary, because factorial and square root are free actions in this ruleset).
      $endgroup$
      – Brilliand
      2 days ago










    • $begingroup$
      @Brilliand in x! ± 1, you never have a divisor lower than x, so subsequent square roots will always have a factor greater than x itself.
      $endgroup$
      – Cœur
      2 days ago










    • $begingroup$
      @Cœur could you add two factorials then modify the result? I would hazard a guess that you could find something that roots to a factor smaller than x. Unlikely whether that would be doable in less than 6 though, more as a counterexample to that proof.
      $endgroup$
      – Riker
      2 days ago








    • 1




      $begingroup$
      Note that you can get 27 = (1+1+1)^(1+1+1) for 6 digits. Just for your illustration!
      $endgroup$
      – CriminallyVulgar
      2 days ago
















    $begingroup$
    Well argued! Can you somehow build a mathematical proof from this line of thinking? It would be great.
    $endgroup$
    – luchonacho
    2 days ago






    $begingroup$
    Well argued! Can you somehow build a mathematical proof from this line of thinking? It would be great.
    $endgroup$
    – luchonacho
    2 days ago






    3




    3




    $begingroup$
    The claim that it's impossible to reach 29 by some iterated factorial, followed by a trivial addition/subtraction, followed by an iterated square root, might be very hard to prove (and necessary, because factorial and square root are free actions in this ruleset).
    $endgroup$
    – Brilliand
    2 days ago




    $begingroup$
    The claim that it's impossible to reach 29 by some iterated factorial, followed by a trivial addition/subtraction, followed by an iterated square root, might be very hard to prove (and necessary, because factorial and square root are free actions in this ruleset).
    $endgroup$
    – Brilliand
    2 days ago












    $begingroup$
    @Brilliand in x! ± 1, you never have a divisor lower than x, so subsequent square roots will always have a factor greater than x itself.
    $endgroup$
    – Cœur
    2 days ago




    $begingroup$
    @Brilliand in x! ± 1, you never have a divisor lower than x, so subsequent square roots will always have a factor greater than x itself.
    $endgroup$
    – Cœur
    2 days ago












    $begingroup$
    @Cœur could you add two factorials then modify the result? I would hazard a guess that you could find something that roots to a factor smaller than x. Unlikely whether that would be doable in less than 6 though, more as a counterexample to that proof.
    $endgroup$
    – Riker
    2 days ago






    $begingroup$
    @Cœur could you add two factorials then modify the result? I would hazard a guess that you could find something that roots to a factor smaller than x. Unlikely whether that would be doable in less than 6 though, more as a counterexample to that proof.
    $endgroup$
    – Riker
    2 days ago






    1




    1




    $begingroup$
    Note that you can get 27 = (1+1+1)^(1+1+1) for 6 digits. Just for your illustration!
    $endgroup$
    – CriminallyVulgar
    2 days ago




    $begingroup$
    Note that you can get 27 = (1+1+1)^(1+1+1) for 6 digits. Just for your illustration!
    $endgroup$
    – CriminallyVulgar
    2 days ago











    10












    $begingroup$

    I managed three 8's:




    $(1+1+1+1)!+(1+1+1)!-1=24+6-1$
    $(1+1+1)!times((1+1+1)!-1)-1=6times(6-1)-1$
    $(11+1+1+1)times(1+1)+1=14times2-1$




    and a very dodgy 6:




    $11!!!!!!!!-(1+1+1+1)=11times3-4$




    and 4:




    $(11-1)!!!!!!!-1=10times3-1$







    share|improve this answer









    $endgroup$









    • 12




      $begingroup$
      ... I don't think that's how factorial works. I'm very sure that is not how factorial works.
      $endgroup$
      – Allan Cao
      Feb 28 at 8:43






    • 9




      $begingroup$
      @AllanCao I believe JonMark Perry is using extended factorial functions. For example, a double factorial is defined to be $n(n-2)(n-4)(n-6)...(3)(1)$ if $n$ is odd, and $n(n-2)(n-4)(n-6)...(4)(2)$ if $n$ is even. If we're calculating a triple-factorial, that evaluates to $n(n-3)(n-6)(n-9)...$. In the case of $(11 - 1)!!!!!!!$ (a seven-factorial), it simplifies like this: $(11 - 1)!!!!!!! = (10)!!!!!!! = (10)(10-7) = (10)(3) = 30$
      $endgroup$
      – Hugh
      Feb 28 at 8:50








    • 2




      $begingroup$
      I see. Since they aren't really factorial (and kind of cheating/cheap), the solution for 6 and 4 don't work.
      $endgroup$
      – Allan Cao
      2 days ago










    • $begingroup$
      multi-factorial, but in most 24-type games they are considered to be a big cheat!!! mind you, so is (1+1).concat(11-1-1). @AllanCao
      $endgroup$
      – JonMark Perry
      2 days ago








    • 2




      $begingroup$
      Which is why the concatenation is only for the original digit.
      $endgroup$
      – Allan Cao
      2 days ago
















    10












    $begingroup$

    I managed three 8's:




    $(1+1+1+1)!+(1+1+1)!-1=24+6-1$
    $(1+1+1)!times((1+1+1)!-1)-1=6times(6-1)-1$
    $(11+1+1+1)times(1+1)+1=14times2-1$




    and a very dodgy 6:




    $11!!!!!!!!-(1+1+1+1)=11times3-4$




    and 4:




    $(11-1)!!!!!!!-1=10times3-1$







    share|improve this answer









    $endgroup$









    • 12




      $begingroup$
      ... I don't think that's how factorial works. I'm very sure that is not how factorial works.
      $endgroup$
      – Allan Cao
      Feb 28 at 8:43






    • 9




      $begingroup$
      @AllanCao I believe JonMark Perry is using extended factorial functions. For example, a double factorial is defined to be $n(n-2)(n-4)(n-6)...(3)(1)$ if $n$ is odd, and $n(n-2)(n-4)(n-6)...(4)(2)$ if $n$ is even. If we're calculating a triple-factorial, that evaluates to $n(n-3)(n-6)(n-9)...$. In the case of $(11 - 1)!!!!!!!$ (a seven-factorial), it simplifies like this: $(11 - 1)!!!!!!! = (10)!!!!!!! = (10)(10-7) = (10)(3) = 30$
      $endgroup$
      – Hugh
      Feb 28 at 8:50








    • 2




      $begingroup$
      I see. Since they aren't really factorial (and kind of cheating/cheap), the solution for 6 and 4 don't work.
      $endgroup$
      – Allan Cao
      2 days ago










    • $begingroup$
      multi-factorial, but in most 24-type games they are considered to be a big cheat!!! mind you, so is (1+1).concat(11-1-1). @AllanCao
      $endgroup$
      – JonMark Perry
      2 days ago








    • 2




      $begingroup$
      Which is why the concatenation is only for the original digit.
      $endgroup$
      – Allan Cao
      2 days ago














    10












    10








    10





    $begingroup$

    I managed three 8's:




    $(1+1+1+1)!+(1+1+1)!-1=24+6-1$
    $(1+1+1)!times((1+1+1)!-1)-1=6times(6-1)-1$
    $(11+1+1+1)times(1+1)+1=14times2-1$




    and a very dodgy 6:




    $11!!!!!!!!-(1+1+1+1)=11times3-4$




    and 4:




    $(11-1)!!!!!!!-1=10times3-1$







    share|improve this answer









    $endgroup$



    I managed three 8's:




    $(1+1+1+1)!+(1+1+1)!-1=24+6-1$
    $(1+1+1)!times((1+1+1)!-1)-1=6times(6-1)-1$
    $(11+1+1+1)times(1+1)+1=14times2-1$




    and a very dodgy 6:




    $11!!!!!!!!-(1+1+1+1)=11times3-4$




    and 4:




    $(11-1)!!!!!!!-1=10times3-1$








    share|improve this answer












    share|improve this answer



    share|improve this answer










    answered Feb 28 at 8:22









    JonMark PerryJonMark Perry

    19.5k63992




    19.5k63992








    • 12




      $begingroup$
      ... I don't think that's how factorial works. I'm very sure that is not how factorial works.
      $endgroup$
      – Allan Cao
      Feb 28 at 8:43






    • 9




      $begingroup$
      @AllanCao I believe JonMark Perry is using extended factorial functions. For example, a double factorial is defined to be $n(n-2)(n-4)(n-6)...(3)(1)$ if $n$ is odd, and $n(n-2)(n-4)(n-6)...(4)(2)$ if $n$ is even. If we're calculating a triple-factorial, that evaluates to $n(n-3)(n-6)(n-9)...$. In the case of $(11 - 1)!!!!!!!$ (a seven-factorial), it simplifies like this: $(11 - 1)!!!!!!! = (10)!!!!!!! = (10)(10-7) = (10)(3) = 30$
      $endgroup$
      – Hugh
      Feb 28 at 8:50








    • 2




      $begingroup$
      I see. Since they aren't really factorial (and kind of cheating/cheap), the solution for 6 and 4 don't work.
      $endgroup$
      – Allan Cao
      2 days ago










    • $begingroup$
      multi-factorial, but in most 24-type games they are considered to be a big cheat!!! mind you, so is (1+1).concat(11-1-1). @AllanCao
      $endgroup$
      – JonMark Perry
      2 days ago








    • 2




      $begingroup$
      Which is why the concatenation is only for the original digit.
      $endgroup$
      – Allan Cao
      2 days ago














    • 12




      $begingroup$
      ... I don't think that's how factorial works. I'm very sure that is not how factorial works.
      $endgroup$
      – Allan Cao
      Feb 28 at 8:43






    • 9




      $begingroup$
      @AllanCao I believe JonMark Perry is using extended factorial functions. For example, a double factorial is defined to be $n(n-2)(n-4)(n-6)...(3)(1)$ if $n$ is odd, and $n(n-2)(n-4)(n-6)...(4)(2)$ if $n$ is even. If we're calculating a triple-factorial, that evaluates to $n(n-3)(n-6)(n-9)...$. In the case of $(11 - 1)!!!!!!!$ (a seven-factorial), it simplifies like this: $(11 - 1)!!!!!!! = (10)!!!!!!! = (10)(10-7) = (10)(3) = 30$
      $endgroup$
      – Hugh
      Feb 28 at 8:50








    • 2




      $begingroup$
      I see. Since they aren't really factorial (and kind of cheating/cheap), the solution for 6 and 4 don't work.
      $endgroup$
      – Allan Cao
      2 days ago










    • $begingroup$
      multi-factorial, but in most 24-type games they are considered to be a big cheat!!! mind you, so is (1+1).concat(11-1-1). @AllanCao
      $endgroup$
      – JonMark Perry
      2 days ago








    • 2




      $begingroup$
      Which is why the concatenation is only for the original digit.
      $endgroup$
      – Allan Cao
      2 days ago








    12




    12




    $begingroup$
    ... I don't think that's how factorial works. I'm very sure that is not how factorial works.
    $endgroup$
    – Allan Cao
    Feb 28 at 8:43




    $begingroup$
    ... I don't think that's how factorial works. I'm very sure that is not how factorial works.
    $endgroup$
    – Allan Cao
    Feb 28 at 8:43




    9




    9




    $begingroup$
    @AllanCao I believe JonMark Perry is using extended factorial functions. For example, a double factorial is defined to be $n(n-2)(n-4)(n-6)...(3)(1)$ if $n$ is odd, and $n(n-2)(n-4)(n-6)...(4)(2)$ if $n$ is even. If we're calculating a triple-factorial, that evaluates to $n(n-3)(n-6)(n-9)...$. In the case of $(11 - 1)!!!!!!!$ (a seven-factorial), it simplifies like this: $(11 - 1)!!!!!!! = (10)!!!!!!! = (10)(10-7) = (10)(3) = 30$
    $endgroup$
    – Hugh
    Feb 28 at 8:50






    $begingroup$
    @AllanCao I believe JonMark Perry is using extended factorial functions. For example, a double factorial is defined to be $n(n-2)(n-4)(n-6)...(3)(1)$ if $n$ is odd, and $n(n-2)(n-4)(n-6)...(4)(2)$ if $n$ is even. If we're calculating a triple-factorial, that evaluates to $n(n-3)(n-6)(n-9)...$. In the case of $(11 - 1)!!!!!!!$ (a seven-factorial), it simplifies like this: $(11 - 1)!!!!!!! = (10)!!!!!!! = (10)(10-7) = (10)(3) = 30$
    $endgroup$
    – Hugh
    Feb 28 at 8:50






    2




    2




    $begingroup$
    I see. Since they aren't really factorial (and kind of cheating/cheap), the solution for 6 and 4 don't work.
    $endgroup$
    – Allan Cao
    2 days ago




    $begingroup$
    I see. Since they aren't really factorial (and kind of cheating/cheap), the solution for 6 and 4 don't work.
    $endgroup$
    – Allan Cao
    2 days ago












    $begingroup$
    multi-factorial, but in most 24-type games they are considered to be a big cheat!!! mind you, so is (1+1).concat(11-1-1). @AllanCao
    $endgroup$
    – JonMark Perry
    2 days ago






    $begingroup$
    multi-factorial, but in most 24-type games they are considered to be a big cheat!!! mind you, so is (1+1).concat(11-1-1). @AllanCao
    $endgroup$
    – JonMark Perry
    2 days ago






    2




    2




    $begingroup$
    Which is why the concatenation is only for the original digit.
    $endgroup$
    – Allan Cao
    2 days ago




    $begingroup$
    Which is why the concatenation is only for the original digit.
    $endgroup$
    – Allan Cao
    2 days ago











    8












    $begingroup$

    According to my analysis, there is no solution using less than 7 digits. The proof would require an exhaustion of every possible operation from 29, reducing the dimensionality of the problem one digit at a time, like a tree. I haven't followed all the leaves of the tree, but it seems clear no one leads to a solution.



    To see how my logic works, consider an example:



    $29 + 1 = 30$



    Here we have a sub problem, which is to find a way to get to 30 using 5 or less digits. You can then follow the leaves of the tree, reducing the dimensionality one at a time, until you see there is no solution. For example, 30 + 1 = 31, and finding 31 from 4 digits, etc. Or 30 + 11 = 41, and so on.



    Importantly, many of the branches of the original problem are identical. For example, $29 + 1 = 30$ is the same as $29+1! = 30$; and the subbranch above $30 + 1 = 31$ is exactly the same as $29+(1+1) = 31$ and $29+(1+1)! = 31$.



    Same logic applies to, for instance, $29-1=28$. You need to get to 28 using 5 digits. Following the branches seem to lead to nowhere.



    Some routes are evidently discarded. For instance, 11! = 39916800, too high. Would need to be scaled with a relatively high number, like 111. But that uses 5 of 6 digits. No way to use the last one meaningfully. Same logic with 111! or other like (11-1)! Other routes, like (1+1+1)!=6, requires you to produce 23 (so 23+6=29) with only 3 digits. Impossible.



    No idea why you would ever consider square root as a relevant operation. Temptation here is that somehow a squared root times a natural number ($sqrt{a}b$) will give a natural number. However, this is only true if $ab^2$ is itself the square of a natural number. So you should start by searching for the latter. For example, take 36. It's the square of 6. Can you find natural numbers a and b such that $ab^2=36$? Yes, a=4 and b=3, or a=9 and b=2. But these use too many digits. Follow down the tree and you are done very quickly. What about a=11? It can be seen that it is not possible to find a natural number b such that $sqrt{11b^2}$ is a natural number.



    The above applies the same with division, e.g. $frac{sqrt{a}}{b}$ or $frac{b}{sqrt{a}}$ routes.



    I haven't given you any proof of my claim and I might be totally wrong. But my methodology (reducing the dimensionality of the problem one at a time) led me to nowhere.






    share|improve this answer











    $endgroup$









    • 3




      $begingroup$
      My solution for 7 digits uses a square root quite nicely.
      $endgroup$
      – Amorydai
      Feb 28 at 15:15










    • $begingroup$
      @Amorydai True. I did overlook something like $sqrt{a+b}$, focusing instead only on multiplicative forms, like $sqrt{ab}$. I just had another go from derivations of yours, and it seems impossible to reduce it 1 digit. I could be wrong! As I said, I cannot offer proof, only failed trials. I do not want to go down every possible branch of the tree, even though it is possible.
      $endgroup$
      – luchonacho
      2 days ago












    • $begingroup$
      The only cases where $sqrt{a}b$ is a natural with both $a$ and $b$ natural are when $a$ is already a square, or when $b=0$.
      $endgroup$
      – user2357112
      2 days ago






    • 4




      $begingroup$
      Your "reducing the dimensionality one at a time" logic fails to account for the possibility of combining two expressions that both use more than one digit. For example, $(1+1+1)(1+1+1)$ cannot be decomposed into a 5-digit expression and a 1-digit expression.
      $endgroup$
      – user2357112
      2 days ago










    • $begingroup$
      @user2357112 True, but to the extent that these expressions have to be multiplied or added to other terms in order to reach 29, they are covered in my examples. For instance, your example gives a 9. That is precisely one of my examples, where $a=9$ and $b=2$. In case of addition/substraction, you still need to get to 20 (29-9) or 38 (29+9) using the remaining digits.
      $endgroup$
      – luchonacho
      2 days ago


















    8












    $begingroup$

    According to my analysis, there is no solution using less than 7 digits. The proof would require an exhaustion of every possible operation from 29, reducing the dimensionality of the problem one digit at a time, like a tree. I haven't followed all the leaves of the tree, but it seems clear no one leads to a solution.



    To see how my logic works, consider an example:



    $29 + 1 = 30$



    Here we have a sub problem, which is to find a way to get to 30 using 5 or less digits. You can then follow the leaves of the tree, reducing the dimensionality one at a time, until you see there is no solution. For example, 30 + 1 = 31, and finding 31 from 4 digits, etc. Or 30 + 11 = 41, and so on.



    Importantly, many of the branches of the original problem are identical. For example, $29 + 1 = 30$ is the same as $29+1! = 30$; and the subbranch above $30 + 1 = 31$ is exactly the same as $29+(1+1) = 31$ and $29+(1+1)! = 31$.



    Same logic applies to, for instance, $29-1=28$. You need to get to 28 using 5 digits. Following the branches seem to lead to nowhere.



    Some routes are evidently discarded. For instance, 11! = 39916800, too high. Would need to be scaled with a relatively high number, like 111. But that uses 5 of 6 digits. No way to use the last one meaningfully. Same logic with 111! or other like (11-1)! Other routes, like (1+1+1)!=6, requires you to produce 23 (so 23+6=29) with only 3 digits. Impossible.



    No idea why you would ever consider square root as a relevant operation. Temptation here is that somehow a squared root times a natural number ($sqrt{a}b$) will give a natural number. However, this is only true if $ab^2$ is itself the square of a natural number. So you should start by searching for the latter. For example, take 36. It's the square of 6. Can you find natural numbers a and b such that $ab^2=36$? Yes, a=4 and b=3, or a=9 and b=2. But these use too many digits. Follow down the tree and you are done very quickly. What about a=11? It can be seen that it is not possible to find a natural number b such that $sqrt{11b^2}$ is a natural number.



    The above applies the same with division, e.g. $frac{sqrt{a}}{b}$ or $frac{b}{sqrt{a}}$ routes.



    I haven't given you any proof of my claim and I might be totally wrong. But my methodology (reducing the dimensionality of the problem one at a time) led me to nowhere.






    share|improve this answer











    $endgroup$









    • 3




      $begingroup$
      My solution for 7 digits uses a square root quite nicely.
      $endgroup$
      – Amorydai
      Feb 28 at 15:15










    • $begingroup$
      @Amorydai True. I did overlook something like $sqrt{a+b}$, focusing instead only on multiplicative forms, like $sqrt{ab}$. I just had another go from derivations of yours, and it seems impossible to reduce it 1 digit. I could be wrong! As I said, I cannot offer proof, only failed trials. I do not want to go down every possible branch of the tree, even though it is possible.
      $endgroup$
      – luchonacho
      2 days ago












    • $begingroup$
      The only cases where $sqrt{a}b$ is a natural with both $a$ and $b$ natural are when $a$ is already a square, or when $b=0$.
      $endgroup$
      – user2357112
      2 days ago






    • 4




      $begingroup$
      Your "reducing the dimensionality one at a time" logic fails to account for the possibility of combining two expressions that both use more than one digit. For example, $(1+1+1)(1+1+1)$ cannot be decomposed into a 5-digit expression and a 1-digit expression.
      $endgroup$
      – user2357112
      2 days ago










    • $begingroup$
      @user2357112 True, but to the extent that these expressions have to be multiplied or added to other terms in order to reach 29, they are covered in my examples. For instance, your example gives a 9. That is precisely one of my examples, where $a=9$ and $b=2$. In case of addition/substraction, you still need to get to 20 (29-9) or 38 (29+9) using the remaining digits.
      $endgroup$
      – luchonacho
      2 days ago
















    8












    8








    8





    $begingroup$

    According to my analysis, there is no solution using less than 7 digits. The proof would require an exhaustion of every possible operation from 29, reducing the dimensionality of the problem one digit at a time, like a tree. I haven't followed all the leaves of the tree, but it seems clear no one leads to a solution.



    To see how my logic works, consider an example:



    $29 + 1 = 30$



    Here we have a sub problem, which is to find a way to get to 30 using 5 or less digits. You can then follow the leaves of the tree, reducing the dimensionality one at a time, until you see there is no solution. For example, 30 + 1 = 31, and finding 31 from 4 digits, etc. Or 30 + 11 = 41, and so on.



    Importantly, many of the branches of the original problem are identical. For example, $29 + 1 = 30$ is the same as $29+1! = 30$; and the subbranch above $30 + 1 = 31$ is exactly the same as $29+(1+1) = 31$ and $29+(1+1)! = 31$.



    Same logic applies to, for instance, $29-1=28$. You need to get to 28 using 5 digits. Following the branches seem to lead to nowhere.



    Some routes are evidently discarded. For instance, 11! = 39916800, too high. Would need to be scaled with a relatively high number, like 111. But that uses 5 of 6 digits. No way to use the last one meaningfully. Same logic with 111! or other like (11-1)! Other routes, like (1+1+1)!=6, requires you to produce 23 (so 23+6=29) with only 3 digits. Impossible.



    No idea why you would ever consider square root as a relevant operation. Temptation here is that somehow a squared root times a natural number ($sqrt{a}b$) will give a natural number. However, this is only true if $ab^2$ is itself the square of a natural number. So you should start by searching for the latter. For example, take 36. It's the square of 6. Can you find natural numbers a and b such that $ab^2=36$? Yes, a=4 and b=3, or a=9 and b=2. But these use too many digits. Follow down the tree and you are done very quickly. What about a=11? It can be seen that it is not possible to find a natural number b such that $sqrt{11b^2}$ is a natural number.



    The above applies the same with division, e.g. $frac{sqrt{a}}{b}$ or $frac{b}{sqrt{a}}$ routes.



    I haven't given you any proof of my claim and I might be totally wrong. But my methodology (reducing the dimensionality of the problem one at a time) led me to nowhere.






    share|improve this answer











    $endgroup$



    According to my analysis, there is no solution using less than 7 digits. The proof would require an exhaustion of every possible operation from 29, reducing the dimensionality of the problem one digit at a time, like a tree. I haven't followed all the leaves of the tree, but it seems clear no one leads to a solution.



    To see how my logic works, consider an example:



    $29 + 1 = 30$



    Here we have a sub problem, which is to find a way to get to 30 using 5 or less digits. You can then follow the leaves of the tree, reducing the dimensionality one at a time, until you see there is no solution. For example, 30 + 1 = 31, and finding 31 from 4 digits, etc. Or 30 + 11 = 41, and so on.



    Importantly, many of the branches of the original problem are identical. For example, $29 + 1 = 30$ is the same as $29+1! = 30$; and the subbranch above $30 + 1 = 31$ is exactly the same as $29+(1+1) = 31$ and $29+(1+1)! = 31$.



    Same logic applies to, for instance, $29-1=28$. You need to get to 28 using 5 digits. Following the branches seem to lead to nowhere.



    Some routes are evidently discarded. For instance, 11! = 39916800, too high. Would need to be scaled with a relatively high number, like 111. But that uses 5 of 6 digits. No way to use the last one meaningfully. Same logic with 111! or other like (11-1)! Other routes, like (1+1+1)!=6, requires you to produce 23 (so 23+6=29) with only 3 digits. Impossible.



    No idea why you would ever consider square root as a relevant operation. Temptation here is that somehow a squared root times a natural number ($sqrt{a}b$) will give a natural number. However, this is only true if $ab^2$ is itself the square of a natural number. So you should start by searching for the latter. For example, take 36. It's the square of 6. Can you find natural numbers a and b such that $ab^2=36$? Yes, a=4 and b=3, or a=9 and b=2. But these use too many digits. Follow down the tree and you are done very quickly. What about a=11? It can be seen that it is not possible to find a natural number b such that $sqrt{11b^2}$ is a natural number.



    The above applies the same with division, e.g. $frac{sqrt{a}}{b}$ or $frac{b}{sqrt{a}}$ routes.



    I haven't given you any proof of my claim and I might be totally wrong. But my methodology (reducing the dimensionality of the problem one at a time) led me to nowhere.







    share|improve this answer














    share|improve this answer



    share|improve this answer








    edited Feb 28 at 12:50

























    answered Feb 28 at 11:53









    luchonacholuchonacho

    2151212




    2151212








    • 3




      $begingroup$
      My solution for 7 digits uses a square root quite nicely.
      $endgroup$
      – Amorydai
      Feb 28 at 15:15










    • $begingroup$
      @Amorydai True. I did overlook something like $sqrt{a+b}$, focusing instead only on multiplicative forms, like $sqrt{ab}$. I just had another go from derivations of yours, and it seems impossible to reduce it 1 digit. I could be wrong! As I said, I cannot offer proof, only failed trials. I do not want to go down every possible branch of the tree, even though it is possible.
      $endgroup$
      – luchonacho
      2 days ago












    • $begingroup$
      The only cases where $sqrt{a}b$ is a natural with both $a$ and $b$ natural are when $a$ is already a square, or when $b=0$.
      $endgroup$
      – user2357112
      2 days ago






    • 4




      $begingroup$
      Your "reducing the dimensionality one at a time" logic fails to account for the possibility of combining two expressions that both use more than one digit. For example, $(1+1+1)(1+1+1)$ cannot be decomposed into a 5-digit expression and a 1-digit expression.
      $endgroup$
      – user2357112
      2 days ago










    • $begingroup$
      @user2357112 True, but to the extent that these expressions have to be multiplied or added to other terms in order to reach 29, they are covered in my examples. For instance, your example gives a 9. That is precisely one of my examples, where $a=9$ and $b=2$. In case of addition/substraction, you still need to get to 20 (29-9) or 38 (29+9) using the remaining digits.
      $endgroup$
      – luchonacho
      2 days ago
















    • 3




      $begingroup$
      My solution for 7 digits uses a square root quite nicely.
      $endgroup$
      – Amorydai
      Feb 28 at 15:15










    • $begingroup$
      @Amorydai True. I did overlook something like $sqrt{a+b}$, focusing instead only on multiplicative forms, like $sqrt{ab}$. I just had another go from derivations of yours, and it seems impossible to reduce it 1 digit. I could be wrong! As I said, I cannot offer proof, only failed trials. I do not want to go down every possible branch of the tree, even though it is possible.
      $endgroup$
      – luchonacho
      2 days ago












    • $begingroup$
      The only cases where $sqrt{a}b$ is a natural with both $a$ and $b$ natural are when $a$ is already a square, or when $b=0$.
      $endgroup$
      – user2357112
      2 days ago






    • 4




      $begingroup$
      Your "reducing the dimensionality one at a time" logic fails to account for the possibility of combining two expressions that both use more than one digit. For example, $(1+1+1)(1+1+1)$ cannot be decomposed into a 5-digit expression and a 1-digit expression.
      $endgroup$
      – user2357112
      2 days ago










    • $begingroup$
      @user2357112 True, but to the extent that these expressions have to be multiplied or added to other terms in order to reach 29, they are covered in my examples. For instance, your example gives a 9. That is precisely one of my examples, where $a=9$ and $b=2$. In case of addition/substraction, you still need to get to 20 (29-9) or 38 (29+9) using the remaining digits.
      $endgroup$
      – luchonacho
      2 days ago










    3




    3




    $begingroup$
    My solution for 7 digits uses a square root quite nicely.
    $endgroup$
    – Amorydai
    Feb 28 at 15:15




    $begingroup$
    My solution for 7 digits uses a square root quite nicely.
    $endgroup$
    – Amorydai
    Feb 28 at 15:15












    $begingroup$
    @Amorydai True. I did overlook something like $sqrt{a+b}$, focusing instead only on multiplicative forms, like $sqrt{ab}$. I just had another go from derivations of yours, and it seems impossible to reduce it 1 digit. I could be wrong! As I said, I cannot offer proof, only failed trials. I do not want to go down every possible branch of the tree, even though it is possible.
    $endgroup$
    – luchonacho
    2 days ago






    $begingroup$
    @Amorydai True. I did overlook something like $sqrt{a+b}$, focusing instead only on multiplicative forms, like $sqrt{ab}$. I just had another go from derivations of yours, and it seems impossible to reduce it 1 digit. I could be wrong! As I said, I cannot offer proof, only failed trials. I do not want to go down every possible branch of the tree, even though it is possible.
    $endgroup$
    – luchonacho
    2 days ago














    $begingroup$
    The only cases where $sqrt{a}b$ is a natural with both $a$ and $b$ natural are when $a$ is already a square, or when $b=0$.
    $endgroup$
    – user2357112
    2 days ago




    $begingroup$
    The only cases where $sqrt{a}b$ is a natural with both $a$ and $b$ natural are when $a$ is already a square, or when $b=0$.
    $endgroup$
    – user2357112
    2 days ago




    4




    4




    $begingroup$
    Your "reducing the dimensionality one at a time" logic fails to account for the possibility of combining two expressions that both use more than one digit. For example, $(1+1+1)(1+1+1)$ cannot be decomposed into a 5-digit expression and a 1-digit expression.
    $endgroup$
    – user2357112
    2 days ago




    $begingroup$
    Your "reducing the dimensionality one at a time" logic fails to account for the possibility of combining two expressions that both use more than one digit. For example, $(1+1+1)(1+1+1)$ cannot be decomposed into a 5-digit expression and a 1-digit expression.
    $endgroup$
    – user2357112
    2 days ago












    $begingroup$
    @user2357112 True, but to the extent that these expressions have to be multiplied or added to other terms in order to reach 29, they are covered in my examples. For instance, your example gives a 9. That is precisely one of my examples, where $a=9$ and $b=2$. In case of addition/substraction, you still need to get to 20 (29-9) or 38 (29+9) using the remaining digits.
    $endgroup$
    – luchonacho
    2 days ago






    $begingroup$
    @user2357112 True, but to the extent that these expressions have to be multiplied or added to other terms in order to reach 29, they are covered in my examples. For instance, your example gives a 9. That is precisely one of my examples, where $a=9$ and $b=2$. In case of addition/substraction, you still need to get to 20 (29-9) or 38 (29+9) using the remaining digits.
    $endgroup$
    – luchonacho
    2 days ago













    2












    $begingroup$

    I can get an answer accurate to within 0.12% with six 1s:




    $$sqrt{sqrt{sqrt{11^{11}}}}+1+1 approx 29.0343$$







    share|improve this answer









    $endgroup$









    • 2




      $begingroup$
      Interesting approach to the question but an exact answer is required.
      $endgroup$
      – Allan Cao
      Feb 28 at 7:25
















    2












    $begingroup$

    I can get an answer accurate to within 0.12% with six 1s:




    $$sqrt{sqrt{sqrt{11^{11}}}}+1+1 approx 29.0343$$







    share|improve this answer









    $endgroup$









    • 2




      $begingroup$
      Interesting approach to the question but an exact answer is required.
      $endgroup$
      – Allan Cao
      Feb 28 at 7:25














    2












    2








    2





    $begingroup$

    I can get an answer accurate to within 0.12% with six 1s:




    $$sqrt{sqrt{sqrt{11^{11}}}}+1+1 approx 29.0343$$







    share|improve this answer









    $endgroup$



    I can get an answer accurate to within 0.12% with six 1s:




    $$sqrt{sqrt{sqrt{11^{11}}}}+1+1 approx 29.0343$$








    share|improve this answer












    share|improve this answer



    share|improve this answer










    answered Feb 28 at 4:53









    jasonharperjasonharper

    460411




    460411








    • 2




      $begingroup$
      Interesting approach to the question but an exact answer is required.
      $endgroup$
      – Allan Cao
      Feb 28 at 7:25














    • 2




      $begingroup$
      Interesting approach to the question but an exact answer is required.
      $endgroup$
      – Allan Cao
      Feb 28 at 7:25








    2




    2




    $begingroup$
    Interesting approach to the question but an exact answer is required.
    $endgroup$
    – Allan Cao
    Feb 28 at 7:25




    $begingroup$
    Interesting approach to the question but an exact answer is required.
    $endgroup$
    – Allan Cao
    Feb 28 at 7:25











    2












    $begingroup$

    Lowest I managed so far is 9 digits:





    • (1 + 1 + 1 + 1)! + 1 + 1 + 1 + 1 + 1;




    • 11*(1 + 1 + 1) - (1 + 1 + 1 + 1)




    Some other ways I came up with:





    • (1 + 1)^(1 + 1 + 1 + 1 + 1) - 1 - 1 - 1 (10 digits)




    • (1 + 1 + 1 + 1 + 1)!/(1 + 1 + 1 + 1) - 1 (10 digits)




    • (1 + 1 + 1 + 1 + 1)^(1 + 1) + 1 + 1 + 1 + 1 (11 digits)




    • 11*(1 + 1) + 1 + 1 + 1 + 1 + 1 + 1 + 1 (11 digits)








    share|improve this answer











    $endgroup$













    • $begingroup$
      Looking at your first 10 and 11 digit solutions, can't you simplify to (1+1+1)^(1+1+1)+1+1 for 8 digits?
      $endgroup$
      – Chronocidal
      Feb 28 at 10:07










    • $begingroup$
      @Chronocidal: Less simplified and more a different solution...
      $endgroup$
      – Chris
      Feb 28 at 14:44
















    2












    $begingroup$

    Lowest I managed so far is 9 digits:





    • (1 + 1 + 1 + 1)! + 1 + 1 + 1 + 1 + 1;




    • 11*(1 + 1 + 1) - (1 + 1 + 1 + 1)




    Some other ways I came up with:





    • (1 + 1)^(1 + 1 + 1 + 1 + 1) - 1 - 1 - 1 (10 digits)




    • (1 + 1 + 1 + 1 + 1)!/(1 + 1 + 1 + 1) - 1 (10 digits)




    • (1 + 1 + 1 + 1 + 1)^(1 + 1) + 1 + 1 + 1 + 1 (11 digits)




    • 11*(1 + 1) + 1 + 1 + 1 + 1 + 1 + 1 + 1 (11 digits)








    share|improve this answer











    $endgroup$













    • $begingroup$
      Looking at your first 10 and 11 digit solutions, can't you simplify to (1+1+1)^(1+1+1)+1+1 for 8 digits?
      $endgroup$
      – Chronocidal
      Feb 28 at 10:07










    • $begingroup$
      @Chronocidal: Less simplified and more a different solution...
      $endgroup$
      – Chris
      Feb 28 at 14:44














    2












    2








    2





    $begingroup$

    Lowest I managed so far is 9 digits:





    • (1 + 1 + 1 + 1)! + 1 + 1 + 1 + 1 + 1;




    • 11*(1 + 1 + 1) - (1 + 1 + 1 + 1)




    Some other ways I came up with:





    • (1 + 1)^(1 + 1 + 1 + 1 + 1) - 1 - 1 - 1 (10 digits)




    • (1 + 1 + 1 + 1 + 1)!/(1 + 1 + 1 + 1) - 1 (10 digits)




    • (1 + 1 + 1 + 1 + 1)^(1 + 1) + 1 + 1 + 1 + 1 (11 digits)




    • 11*(1 + 1) + 1 + 1 + 1 + 1 + 1 + 1 + 1 (11 digits)








    share|improve this answer











    $endgroup$



    Lowest I managed so far is 9 digits:





    • (1 + 1 + 1 + 1)! + 1 + 1 + 1 + 1 + 1;




    • 11*(1 + 1 + 1) - (1 + 1 + 1 + 1)




    Some other ways I came up with:





    • (1 + 1)^(1 + 1 + 1 + 1 + 1) - 1 - 1 - 1 (10 digits)




    • (1 + 1 + 1 + 1 + 1)!/(1 + 1 + 1 + 1) - 1 (10 digits)




    • (1 + 1 + 1 + 1 + 1)^(1 + 1) + 1 + 1 + 1 + 1 (11 digits)




    • 11*(1 + 1) + 1 + 1 + 1 + 1 + 1 + 1 + 1 (11 digits)









    share|improve this answer














    share|improve this answer



    share|improve this answer








    edited Feb 28 at 7:40









    Rai

    995112




    995112










    answered Feb 28 at 2:30









    simonzacksimonzack

    294110




    294110












    • $begingroup$
      Looking at your first 10 and 11 digit solutions, can't you simplify to (1+1+1)^(1+1+1)+1+1 for 8 digits?
      $endgroup$
      – Chronocidal
      Feb 28 at 10:07










    • $begingroup$
      @Chronocidal: Less simplified and more a different solution...
      $endgroup$
      – Chris
      Feb 28 at 14:44


















    • $begingroup$
      Looking at your first 10 and 11 digit solutions, can't you simplify to (1+1+1)^(1+1+1)+1+1 for 8 digits?
      $endgroup$
      – Chronocidal
      Feb 28 at 10:07










    • $begingroup$
      @Chronocidal: Less simplified and more a different solution...
      $endgroup$
      – Chris
      Feb 28 at 14:44
















    $begingroup$
    Looking at your first 10 and 11 digit solutions, can't you simplify to (1+1+1)^(1+1+1)+1+1 for 8 digits?
    $endgroup$
    – Chronocidal
    Feb 28 at 10:07




    $begingroup$
    Looking at your first 10 and 11 digit solutions, can't you simplify to (1+1+1)^(1+1+1)+1+1 for 8 digits?
    $endgroup$
    – Chronocidal
    Feb 28 at 10:07












    $begingroup$
    @Chronocidal: Less simplified and more a different solution...
    $endgroup$
    – Chris
    Feb 28 at 14:44




    $begingroup$
    @Chronocidal: Less simplified and more a different solution...
    $endgroup$
    – Chris
    Feb 28 at 14:44











    1












    $begingroup$

    For the sake of completeness, here's an 8 digit solution:




    (1+1+1)^(1+1+1) + 1 + 1







    share|improve this answer









    $endgroup$


















      1












      $begingroup$

      For the sake of completeness, here's an 8 digit solution:




      (1+1+1)^(1+1+1) + 1 + 1







      share|improve this answer









      $endgroup$
















        1












        1








        1





        $begingroup$

        For the sake of completeness, here's an 8 digit solution:




        (1+1+1)^(1+1+1) + 1 + 1







        share|improve this answer









        $endgroup$



        For the sake of completeness, here's an 8 digit solution:




        (1+1+1)^(1+1+1) + 1 + 1








        share|improve this answer












        share|improve this answer



        share|improve this answer










        answered Feb 28 at 3:43









        kwypstonkwypston

        1863




        1863























            1












            $begingroup$

            This was posted before the question was edited to demand an integer, and requires that the rules accept rounding, but it does produce the digits 29. Six is achievable in this way:




            $sqrt{( ( 1 + 1 + 1 )! + 1 )} * 11 approx 29.1033$







            share|improve this answer











            $endgroup$









            • 2




              $begingroup$
              Interesting approach to the question but an exact answer is required.
              $endgroup$
              – Allan Cao
              Feb 28 at 7:25










            • $begingroup$
              You could fix it with the floor: $lfloorsqrt{( ( 1 + 1 + 1 )! + 1 )} * 11rfloor = 29$
              $endgroup$
              – Dr Xorile
              2 days ago












            • $begingroup$
              Floor can't be used. As well, I just assumed that people would take 29 as exactly 29 not around 29.
              $endgroup$
              – Allan Cao
              2 days ago










            • $begingroup$
              Well, as the problem originally stated "make 29" without an explicit restriction to an exact integer result, asking on a puzzling site where people enjoy posting creative/stretch answers will bring one to accept such responses. Before the restriction was imposed, the multiple definitions of "make" might not rule out extra digits appearing.
              $endgroup$
              – a sandwhich
              yesterday


















            1












            $begingroup$

            This was posted before the question was edited to demand an integer, and requires that the rules accept rounding, but it does produce the digits 29. Six is achievable in this way:




            $sqrt{( ( 1 + 1 + 1 )! + 1 )} * 11 approx 29.1033$







            share|improve this answer











            $endgroup$









            • 2




              $begingroup$
              Interesting approach to the question but an exact answer is required.
              $endgroup$
              – Allan Cao
              Feb 28 at 7:25










            • $begingroup$
              You could fix it with the floor: $lfloorsqrt{( ( 1 + 1 + 1 )! + 1 )} * 11rfloor = 29$
              $endgroup$
              – Dr Xorile
              2 days ago












            • $begingroup$
              Floor can't be used. As well, I just assumed that people would take 29 as exactly 29 not around 29.
              $endgroup$
              – Allan Cao
              2 days ago










            • $begingroup$
              Well, as the problem originally stated "make 29" without an explicit restriction to an exact integer result, asking on a puzzling site where people enjoy posting creative/stretch answers will bring one to accept such responses. Before the restriction was imposed, the multiple definitions of "make" might not rule out extra digits appearing.
              $endgroup$
              – a sandwhich
              yesterday
















            1












            1








            1





            $begingroup$

            This was posted before the question was edited to demand an integer, and requires that the rules accept rounding, but it does produce the digits 29. Six is achievable in this way:




            $sqrt{( ( 1 + 1 + 1 )! + 1 )} * 11 approx 29.1033$







            share|improve this answer











            $endgroup$



            This was posted before the question was edited to demand an integer, and requires that the rules accept rounding, but it does produce the digits 29. Six is achievable in this way:




            $sqrt{( ( 1 + 1 + 1 )! + 1 )} * 11 approx 29.1033$








            share|improve this answer














            share|improve this answer



            share|improve this answer








            edited 2 days ago

























            answered Feb 28 at 4:51









            a sandwhicha sandwhich

            46917




            46917








            • 2




              $begingroup$
              Interesting approach to the question but an exact answer is required.
              $endgroup$
              – Allan Cao
              Feb 28 at 7:25










            • $begingroup$
              You could fix it with the floor: $lfloorsqrt{( ( 1 + 1 + 1 )! + 1 )} * 11rfloor = 29$
              $endgroup$
              – Dr Xorile
              2 days ago












            • $begingroup$
              Floor can't be used. As well, I just assumed that people would take 29 as exactly 29 not around 29.
              $endgroup$
              – Allan Cao
              2 days ago










            • $begingroup$
              Well, as the problem originally stated "make 29" without an explicit restriction to an exact integer result, asking on a puzzling site where people enjoy posting creative/stretch answers will bring one to accept such responses. Before the restriction was imposed, the multiple definitions of "make" might not rule out extra digits appearing.
              $endgroup$
              – a sandwhich
              yesterday
















            • 2




              $begingroup$
              Interesting approach to the question but an exact answer is required.
              $endgroup$
              – Allan Cao
              Feb 28 at 7:25










            • $begingroup$
              You could fix it with the floor: $lfloorsqrt{( ( 1 + 1 + 1 )! + 1 )} * 11rfloor = 29$
              $endgroup$
              – Dr Xorile
              2 days ago












            • $begingroup$
              Floor can't be used. As well, I just assumed that people would take 29 as exactly 29 not around 29.
              $endgroup$
              – Allan Cao
              2 days ago










            • $begingroup$
              Well, as the problem originally stated "make 29" without an explicit restriction to an exact integer result, asking on a puzzling site where people enjoy posting creative/stretch answers will bring one to accept such responses. Before the restriction was imposed, the multiple definitions of "make" might not rule out extra digits appearing.
              $endgroup$
              – a sandwhich
              yesterday










            2




            2




            $begingroup$
            Interesting approach to the question but an exact answer is required.
            $endgroup$
            – Allan Cao
            Feb 28 at 7:25




            $begingroup$
            Interesting approach to the question but an exact answer is required.
            $endgroup$
            – Allan Cao
            Feb 28 at 7:25












            $begingroup$
            You could fix it with the floor: $lfloorsqrt{( ( 1 + 1 + 1 )! + 1 )} * 11rfloor = 29$
            $endgroup$
            – Dr Xorile
            2 days ago






            $begingroup$
            You could fix it with the floor: $lfloorsqrt{( ( 1 + 1 + 1 )! + 1 )} * 11rfloor = 29$
            $endgroup$
            – Dr Xorile
            2 days ago














            $begingroup$
            Floor can't be used. As well, I just assumed that people would take 29 as exactly 29 not around 29.
            $endgroup$
            – Allan Cao
            2 days ago




            $begingroup$
            Floor can't be used. As well, I just assumed that people would take 29 as exactly 29 not around 29.
            $endgroup$
            – Allan Cao
            2 days ago












            $begingroup$
            Well, as the problem originally stated "make 29" without an explicit restriction to an exact integer result, asking on a puzzling site where people enjoy posting creative/stretch answers will bring one to accept such responses. Before the restriction was imposed, the multiple definitions of "make" might not rule out extra digits appearing.
            $endgroup$
            – a sandwhich
            yesterday






            $begingroup$
            Well, as the problem originally stated "make 29" without an explicit restriction to an exact integer result, asking on a puzzling site where people enjoy posting creative/stretch answers will bring one to accept such responses. Before the restriction was imposed, the multiple definitions of "make" might not rule out extra digits appearing.
            $endgroup$
            – a sandwhich
            yesterday













            0












            $begingroup$


            11+11+11-1-1-1-1=29 This is my best guess.







            share|improve this answer










            New contributor




            Jodi Ansley is a new contributor to this site. Take care in asking for clarification, commenting, and answering.
            Check out our Code of Conduct.






            $endgroup$









            • 7




              $begingroup$
              Even 11 * (1+1+1) is better for the beginning.
              $endgroup$
              – Allan Cao
              Feb 28 at 7:22
















            0












            $begingroup$


            11+11+11-1-1-1-1=29 This is my best guess.







            share|improve this answer










            New contributor




            Jodi Ansley is a new contributor to this site. Take care in asking for clarification, commenting, and answering.
            Check out our Code of Conduct.






            $endgroup$









            • 7




              $begingroup$
              Even 11 * (1+1+1) is better for the beginning.
              $endgroup$
              – Allan Cao
              Feb 28 at 7:22














            0












            0








            0





            $begingroup$


            11+11+11-1-1-1-1=29 This is my best guess.







            share|improve this answer










            New contributor




            Jodi Ansley is a new contributor to this site. Take care in asking for clarification, commenting, and answering.
            Check out our Code of Conduct.






            $endgroup$




            11+11+11-1-1-1-1=29 This is my best guess.








            share|improve this answer










            New contributor




            Jodi Ansley is a new contributor to this site. Take care in asking for clarification, commenting, and answering.
            Check out our Code of Conduct.









            share|improve this answer



            share|improve this answer








            edited Feb 28 at 7:33









            rhsquared

            8,18031849




            8,18031849






            New contributor




            Jodi Ansley is a new contributor to this site. Take care in asking for clarification, commenting, and answering.
            Check out our Code of Conduct.









            answered Feb 28 at 7:10









            Jodi AnsleyJodi Ansley

            21




            21




            New contributor




            Jodi Ansley is a new contributor to this site. Take care in asking for clarification, commenting, and answering.
            Check out our Code of Conduct.





            New contributor





            Jodi Ansley is a new contributor to this site. Take care in asking for clarification, commenting, and answering.
            Check out our Code of Conduct.






            Jodi Ansley is a new contributor to this site. Take care in asking for clarification, commenting, and answering.
            Check out our Code of Conduct.








            • 7




              $begingroup$
              Even 11 * (1+1+1) is better for the beginning.
              $endgroup$
              – Allan Cao
              Feb 28 at 7:22














            • 7




              $begingroup$
              Even 11 * (1+1+1) is better for the beginning.
              $endgroup$
              – Allan Cao
              Feb 28 at 7:22








            7




            7




            $begingroup$
            Even 11 * (1+1+1) is better for the beginning.
            $endgroup$
            – Allan Cao
            Feb 28 at 7:22




            $begingroup$
            Even 11 * (1+1+1) is better for the beginning.
            $endgroup$
            – Allan Cao
            Feb 28 at 7:22





            protected by Rubio Feb 28 at 7:46



            Thank you for your interest in this question.
            Because it has attracted low-quality or spam answers that had to be removed, posting an answer now requires 10 reputation on this site (the association bonus does not count).



            Would you like to answer one of these unanswered questions instead?



            Popular posts from this blog

            Why does my Macbook overheat and use so much CPU and energy when on YouTube?Why do so many insist on using...

            How to prevent page numbers from appearing on glossaries?How to remove a dot and a page number in the...

            Puerta de Hutt Referencias Enlaces externos Menú de navegación15°58′00″S 5°42′00″O /...